Jump to content

Wikipedia:Reference desk/Science

From Wikipedia, the free encyclopedia

This is an old revision of this page, as edited by 121.221.231.100 (talk) at 03:25, 14 February 2013 (→‎KWh problem). The present address (URL) is a permanent link to this revision, which may differ significantly from the current revision.

Welcome to the science section
of the Wikipedia reference desk.
Select a section:
Want a faster answer?

Main page: Help searching Wikipedia

   

How can I get my question answered?

  • Select the section of the desk that best fits the general topic of your question (see the navigation column to the right).
  • Post your question to only one section, providing a short header that gives the topic of your question.
  • Type '~~~~' (that is, four tilde characters) at the end – this signs and dates your contribution so we know who wrote what and when.
  • Don't post personal contact information – it will be removed. Any answers will be provided here.
  • Please be as specific as possible, and include all relevant context – the usefulness of answers may depend on the context.
  • Note:
    • We don't answer (and may remove) questions that require medical diagnosis or legal advice.
    • We don't answer requests for opinions, predictions or debate.
    • We don't do your homework for you, though we'll help you past the stuck point.
    • We don't conduct original research or provide a free source of ideas, but we'll help you find information you need.



How do I answer a question?

Main page: Wikipedia:Reference desk/Guidelines

  • The best answers address the question directly, and back up facts with wikilinks and links to sources. Do not edit others' comments and do not give any medical or legal advice.
See also:


February 9

Organs Questions

I've got three questions about organs and whatnot?

  • 1. If someone old dies and then an organ/body part of his/hers gets put into another body, does this organ/body part still function based on its actual age or does being put in a new body makes it function as if it were of a younger age?
  • 2. What is the age limit for donating blood, bone marrow, and various organs? Can a 100 year old's donated organ(s) or blood work successfully/well in another person's body?
  • 3. Does any country maintain a list of the blood types and organ donation status of all or most of its population? I'm tempted to think No and that it would be considered a violation of privacy to do something like this, but I am still interested in finding this out for sure.

Thank you very much. Futurist110 (talk) 00:24, 9 February 2013 (UTC)[reply]

1. Yes to the former, age is not a state of mind, it is a biological fact, you can't rejuvenate organs in that way. Plasmic Physics (talk) 00:33, 9 February 2013 (UTC)[reply]
Thank you very much. Here's another question--do the various human organs age and decline in the same way that humans do? For instance, a healthy human who takes care of himself/herself would probably reach age 85 or 90, but afterwards it becomes a very uphill climb, with very few people who reach age 90 reaching age 100 and very few people who reach age 100 reaching age 110. Do most human organs (if put in a functioning body) also stop functioning when they (meaning the organs) are 85-110 years old? Futurist110 (talk) 00:42, 9 February 2013 (UTC)[reply]
I'm not an expert in this this field, but I would say yes, which is why younger transplants are prefered - they have a longer useful lifetime. Plasmic Physics (talk) 00:51, 9 February 2013 (UTC)[reply]
Thank you. I'm interested in this because I saw this article from 2008 about a 123-year-old living cornea (here is that article--http://www.reuters.com/article/2008/10/23/us-norway-eye-idUSTRE49M6BL20081023). Futurist110 (talk) 01:38, 9 February 2013 (UTC)[reply]
That's believable, because I know for certain that different organs age at a different rate. Take a car for example, the tires get worn out faster than the transmission (usually). Plasmic Physics (talk) 01:47, 9 February 2013 (UTC)[reply]
But this contradicts your previous statement of appearing to agree with me that human organs probably age as fast as human themselves do. That said, theoretically it might be possible for a human male to reach age 123, but this hasn't happened before since by that point it would be extremely rare for all of a human male's organs to still be working/alive. Futurist110 (talk) 02:10, 9 February 2013 (UTC)[reply]
I never stated that they age at the same rate, only that they age. Otherwise, please name the time stamp of my comment which is contrary. Plasmic Physics (talk) 05:59, 10 February 2013 (UTC)[reply]
In the UK the age limit for blood donation used to be 70, but they scrapped that (ref), and it's now based on the donor's health. Crucially, certain medications for chronic conditions (medications that are very commonly taken by people as they age) disqualify people as donors, so in practice the number of donors older than 70 is pretty low. -- Finlay McWalterTalk 00:38, 9 February 2013 (UTC)[reply]
So in the U.K. someone aged 100 or above but in good health can (sometimes) donate his/her blood? What about organ donations and bone marrow donations? Futurist110 (talk) 00:42, 9 February 2013 (UTC)[reply]
83 year old kidney donor. 146.90.50.59 (talk) 01:49, 9 February 2013 (UTC)[reply]
Thank you. Futurist110 (talk) 02:10, 9 February 2013 (UTC)[reply]
As I thought I had remembered, and it's confirmed in Red blood cell, blood is constantly being created and recycled, so it might be that any donation age limits are concerns about the general well-being of the donor, rather than any concerns about "old" blood. ←Baseball Bugs What's up, Doc? carrots06:13, 9 February 2013 (UTC)[reply]
When DFDBA (demineralized freeze-dried bone allograft) is procured from cadavers, a molecule known as BMP (bone morphogenetic protein) acts to trigger greater osteblastic potential in the recipient site, theoretically leading to greater bone growth following grafting. The BMPs are even able to trigger de novo bone growth even in the absence of osteblasts by stimulating regular fibroblasts to transform, and studies have shown ossicle formation in murine muscle pouches (in the absence of bone tissue). Anyway, it was shown by Schwartz in 1996 (J Perio) that the age of the cadaver donor correlated with the osteoinductive potential of the bone graft because of the relative potency of the BMPs. He studied the results of DFDBA from both genders and various ages and determined that the dentist (or other clinician) will not know the effects because he or she won't be able to pick the age of the donor. DRosenbach (Talk | Contribs) 15:11, 10 February 2013 (UTC)[reply]

Particle-wave duality

In an atom, it is said that the electron has a probability of being located at a position. Is that technically, or heuristically, correct - is the electron truely whizzing about the atom in a random way, or is it everywhere around the atom at once? I just find it hard to imagine that the electron retains its particle identity. I consider an unbound electron to be like an icy small solar system body, and a bound electron to be like the same SSSB vaporised and converted to the atmosphere of a planet. So, does the probability indicate how often the electron is expected to be encountered at that location, or how much of the electron is to be encountered at that location? Plasmic Physics (talk) 00:30, 9 February 2013 (UTC)[reply]

It's dangerous to think in those "normal" terms. It's not particularly meaningful to ask where the electron "is" or how much of it is where - because it's really only a probability field - and it (in a sense) "teleports" between locations within the probability cloud. For example, there is an effect called "Quantum tunnelling" (which is what makes the flash memory in your phone/tablet/memory-stick work - so we know it's true!). In very simple terms, the flash memory cell has a barrier between two locations that (classically) the electron cannot cross - but it can teleport between those two locations if properly coerced because there is always a finite probability of it being on the other side of the uncrossable barrier! So this is a very real effect - large, real-world things like telephones rely utterly on this weird quantum behavior. But it's quite hard to reconcile with day to day life...and that's the core problem with understanding quantum theory. At those levels of existence, things are very, very weird - and trying to get your head around a physical understanding of it is impossible. The only real way to get a handle on it is via the math. SteveBaker (talk) 01:02, 9 February 2013 (UTC)[reply]
So, a bound electron has a precise location that changes with time? I know about quantum teleportation, I just never reconcilled it with the behaviour of bound electrons. They never discussed electron motion within atoms in the lectures. Plasmic Physics (talk) 01:42, 9 February 2013 (UTC)[reply]
Quick aside — what's being discussed is quantum tunneling, not quantum teleportation, which I think is some information-theoretic abstraction rather than an electron actually showing up in a different place.
I figured. Plasmic Physics (talk) 03:48, 9 February 2013 (UTC)[reply]
Whether an electron has a precise location is a subtle question that gets into interpretations of quantum mechanics. I think most physicists generally prefer to avoid the question altogether ("shut up and calculate" says the shade of Feynman) and leave it to the philosophers. --Trovatore (talk) 03:22, 9 February 2013 (UTC)[reply]
Is there a way of testing either alternative? Plasmic Physics (talk) 03:48, 9 February 2013 (UTC)[reply]
There are more than two alternatives. As far as I know, no experiments have been devised that would distinguish among any of them. Strict positivists probably consider it a meaningless question for that reason, but for people with a more realistic view this is hard to swallow. But it's hard to get realism to play nice with QM under any conditions. --Trovatore (talk) 04:03, 9 February 2013 (UTC)[reply]
I should say, though, that your notion of a sort of "electron vapor", spread out over space, is not really one of the alternatives. Or at least I don't think it is. The electron itself is very tiny — as far as anyone knows, pointlike. The probability distribution tells you how likely it is that the electron will be found at a given location, but it is not the electron itself. --Trovatore (talk) 05:12, 9 February 2013 (UTC)[reply]
The thing is, the "wave" vs. "particle" thing is a false dichotomy. There are not "alternatives" and these are not the two. Electrons always behave like electrons. The problem is that the way they behave simply does not have analogues you can experience with your 5 senses. That is, there is nothing in anything you have experienced, and thus nothing you can "visualize" which fully captures what an electron is. The best we can say is that, in some applications it is helpful to visualize the electron as a particle (but only for those applications) and for other applications, it is helpful to visualize the electron as a wave. Those are still human-created models, however. The electron doesn't change its behavior, it doesn't swap "modes" or jump between "alternatives": it just goes on being an electron doing what electrons always do. It's our problem to come up with models to explain it. And it's been about a century since physicists stopped trying to create a "picture" of what an electron is. That's the whole spirit of the "shut up and calculate" exhortation is above. Electrons are best modeled by mathematical equations that describe and predict what they do: that there is no single nice visualization you can make which captures the full nature of "electronness" is ultimately not the electron's problem. --Jayron32 05:22, 10 February 2013 (UTC)[reply]
The picture of the electron in a hydrogen atom as a stationary wave and the picture of it as a particle randomly moving around are about equally valid. They can't be tested against each other—they're just two different ways of looking at the same physics.
Quantum mechanics actually has two different classical limits. If you take ħ → 0 while keeping the E in E=hf constant (so that f → ∞) you get the classical particle limit: each particle still carries energy E, but the infinite wavelength means you can never observe any wave interference behavior. If you take ħ → 0 while keeping the f in E=hf constant (so that E → 0) you get the classical wave limit: there are infinitely many particles each carrying infinitesimal energy that smoothly cover everything so you never see shot noise. Quite a lot of behavior that's often described as "purely quantum" is really behavior that vanishes in the particle limit but survives in the wave limit. Tunneling is one example of that: in the classical context it's called Evanescent-wave coupling. Of course there are other things that vanish in the wave limit but survive in the particle limit, like shot noise. Quantum mechanics sort of sits halfway between these two types of classical theories, so it's helpful to keep both pictures in mind. -- BenRG (talk) 05:15, 9 February 2013 (UTC)[reply]
I'm worried that this answer sounds overly compatible with the "electron vapor" idea. A stationary wave is not the same as a spread-out electron, any more than a radio wave is a spread-out photon. --Trovatore (talk) 05:26, 9 February 2013 (UTC)[reply]
A radio wave containing one quantum of energy is a spread out photon. A radio wave containing many times that much energy is a bunch of spread out photons. Photons are not localized in a coherent wave.
I'm not sure exactly what you mean by the electron vapor idea but I think there's nothing especially wrong with that idea. Feynman famously said that the single most informative statement about the world is "all things are made of atoms — little particles that move around in perpetual motion, attracting each other when they are a little distance apart, but repelling upon being squeezed into one another". That's a picture of atoms as somewhat sticky, somewhat rubbery balls. Sure they're quantum objects but that doesn't mean these aren't useful classical analogues for various aspects of their behavior. -- BenRG (talk) 06:56, 9 February 2013 (UTC)[reply]
Well, no, come on, I know you're a physicist and I'm not, but a radio wave is not a spread-out photon. A radio wave is electric and magnetic fields. An electron "wave" is an area of probability density. It's not a particle spread out. That's a cheap way of restoring realism that doesn't work.
You can restore realism with the many-worlds interpretation, by letting the real thing be the entire collection of worlds rather than just ours. Maybe you can restore it by the transactional interpretation; I've never understood it well enough to be sure. But not with the image of a "spread-out particle". It's just the wrong image; it doesn't lead to intuitions that correspond with theory or experiment. --Trovatore (talk) 09:12, 9 February 2013 (UTC)[reply]
I'm not a physicist, I just studied it in college. First, as I mentioned in another thread, the Standard Model is a quantized classical field theory. The wave nature of the electromagnetic and electron fields (the latter has no good name) is already there at the classical level. Quantization adds particle-like behavior, but the wavelike behavior doesn't disappear. You can and should think of a photon as the dimmest possible electromagnetic field. It's a misleading name for it, but it's too late to change that. If you want to talk about the pointlike behavior of light (i.e., shot noise), it would probably be better to talk about the "darkenon", which is one quantum of energy transferred from the electromagnetic field to a single silver halide crystal. (I just made that word up.)
Quantization is always the same. You probably wouldn't say that phonons are points and the bulk vibration of the crystal is just an indication of the probability of finding one in a particular place, but it's just as true for them as it is for electrons or photons. Or quantum vortices, which don't seem to have a commonly used name ending in -on but are particles nonetheless. -- BenRG (talk) 18:32, 9 February 2013 (UTC)[reply]
A radio wave totally is just a spread-out photon (or, more commonly, for normal amplitudes, it is a superposition of many photons). It's rarely useful to analyze this way; and we rarely have a wave at RF frequencies whose amplitude is so low as to be a single quantum emission; but this is still definitionally true and is easier to build a single RF photon emitter, in practice, than a single-photon-emitter at optical frequencies. Any electromagnetic wave can be viewed as a wave or as a particle. Considering a macroscopic wavelength as a "particle" can be very instructive in developing intuition about wave-particle duality. BenRG's comments about phonons are spot-on. Nimur (talk) 20:51, 9 February 2013 (UTC)[reply]
I disagree. It's a bad guide to intuition. It avoids confronting the failure of local realism. It sounds as though there's all this real "stuff" that's spread out over space, and that just isn't true. Also it completely clouds (ha!) the issue when you try to extend it to non-pointlike particles, like the proton — the learner can be forgiven for saying, wait a minute, here you're telling me about the proton "radius" (as distinct from the electron radius of, presumably, zero), but at the same time you have a picture of the proton as spread out over space, are these the same thing or not?
Particles are ipso facto localized, but the place they're localized to is, depending on interpretation, uncertain, different in different "worlds", or something else. --Trovatore (talk) 21:00, 9 February 2013 (UTC)[reply]
Well, I mean, the world is as real as it ever was, and failing to acknowledge that seems like an error to me. Of course there's stuff out there; it's what we've been trying to understand with all this physics nonsense. Saying that we've discovered by studying it that it isn't real, aside from being philosophically silly, doesn't tell you anything. It's wakalixes. What Feynman said about atoms does tell you things. The stickiness of macroscopic objects derives from the stickiness of atoms. Solids tend not to be sticky because they have rough surfaces and there's very little atom-to-atom contact when you press them together. Liquids stick to solids because they flow into the gaps. Gas molecules don't stick to each other because they're moving too fast, but if they slow down enough they do stick together, and that's why gases liquefy when you cool them down. And so on. Atoms are solid objects. Macroscopic solids are made of atoms and derive their solidity from the solidity of atoms (which is due to the Pauli exclusion principle). It's ridiculous to deny all of this or to imply that there's some kind of connection to many-worlds or what have you. We've actually learned some things about the everyday world in the last hundred years.
It's true that protons have an intrinsic (interaction) radius and also are spread out and that those things are independent. Saying that you shouldn't tell people about that because it's confusing isn't a very good argument. Mathematically the interaction radius shows up in the interaction part of the Lagrangian as a coupling of field values at different points in space. The photon-electron interaction in QED is a product of field values separately at each space(time) point, integrated over all the points, and that's the sense in which they are pointlike. A free field/particle is not pointlike in any way. That's why I said above that it's probably better to treat the interactions as pointlike rather than the particles themselves.
In many ways Standard Model particles/fields behave like classical particles. But saying that the particle picture is more fundamental, and the wave picture secondary, is completely wrong. -- BenRG (talk) 22:17, 9 February 2013 (UTC)[reply]
Wait a minute, I didn't say there wasn't real stuff out there. I said a delocalized pointlike particle is not "real stuff spread out". I stand by that. A point particle is a point, no matter how spread out its probability density function is. --Trovatore (talk) 22:27, 9 February 2013 (UTC)[reply]
Okay, look. What you seem to be saying is that fundamental particles don't occupy space—which means that the term "occupying space" is useless, since nothing does it—but that they do something, presumably involving the Pauli exclusion principle, that creates the impression of occupying space. I suggest calling that thing that they do "occupying space". I don't see what other meaning the term can have.
As far as I can tell, you haven't explained why you think the particles are just points even in the face of all the arguments to the contrary that I've presented. If it's because you think the wave function is a probability density, I think you're forgetting that it's a probability distribution over classical field configurations, not classical particle coordinates, since the Standard Model is a quantized field theory. If it's because of Feynman diagrams, I'll explain the reasons why Feynman diagrams are unlikely to be fundamental. -- BenRG (talk) 00:04, 10 February 2013 (UTC)[reply]
Well yeah, the path integral formulation makes a lot of sense to me. Why do you see that as non-fundamental?
But you know, anyway, your point about a probability distriubtion over field configurations — that's just a different basis for the vector space. Surely the appropriate basis when you're calling them particles, specifically, is the one where the position distribution is a delta function. That's kind of what "particle" means. --Trovatore (talk) 01:36, 10 February 2013 (UTC)[reply]
To particle physicists "particle" and "field" are nearly synonymous. You're taking the terminology too literally. Atoms aren't indivisible, protons aren't fundamental and particles aren't points. Also, the wave function has an independent value at all field configurations, not just a set of basis states. There's no basis for the quantum Hilbert space that limits you to delta functions in physical space.
Individual Feynman diagrams break gauge symmetry and in some cases involve fictitious particles. The symmetry is restored and the fake particles disappear in the sum, but it's not very plausible that the universe is really adding a bunch of asymmetric diagrams with extra particles and ending up with a perfect symmetry. It's similar to treating the expansion as fundamental. Very similar, in fact, because Feynman diagrams label terms in a series expansion of the path integral. The basic idea is easy to understand. Start with . If H is not a function of time, the solutions are . You can expand that as , i.e., as a sum of ψ0, ψ0 acted on once by −iHt, ψ0 acted on twice by −iHt, .... If H = H1 + H2 then the sum includes ψ0 acted on once by −iH1t, once by −iH2t, twice by −iH1t, once by each, etc. This is very sloppy since I should be using the Lagrangian and I should separate the interacting and noninteracting portions, but I hope you see that in a series expansion you quite naturally get a sum of all finite combinations of discrete interaction terms. That's not to say that the boson-exchange picture suggested by the simplest diagrams is wrong. At least in QED scattering it's correct inasmuch as it's a good approximation, but it's not The Truth any more than E = ½mv².
Another problem is that there are phenomena in the Standard Model that don't have an expansion in Feynman diagrams, such as the Higgs mechanism. (Which is not the same as the Higgs particle, but I think the detection of a particle so similar to the predicted one proves the correctness of the Higgs mechanism beyond a reasonable doubt.)
Another thing is that you can do classical wave mechanics with Feynman diagrams, but there is no particle-like behavior in those theories. -- BenRG (talk) 02:46, 10 February 2013 (UTC)[reply]
'Electron vapour' is exactly the name I would give it - a bound electron completely evaporates, from a particle into a vapour, surrounding the nucleus where the cloud density is equal to the 'probability'. Plasmic Physics (talk) 09:18, 9 February 2013 (UTC)[reply]
But that's exactly the wrong image. If you use that image, your intuition will lead you to incorrect conclusions. The electron has (as far as anyone knows) zero volume — it's just a point. That's true whether it's bound or free.
The "cloud" is a quantum superposition of places the electron might be. But each of those places is just a point (again, as far as anyone knows). --Trovatore (talk) 09:24, 9 February 2013 (UTC)[reply]
Yes, that's wrong—the electron's nature doesn't change just because it's part of an atom. If it's a cloud in that situation, it's a cloud the rest of the time too. If it's a point particle the rest of the time, it's a point particle when part of an atom. -- BenRG (talk) 18:32, 9 February 2013 (UTC)[reply]
There are several mistakes here. You're forgetting the solid-illusion, nothing is truely solid, especially electrons. Electrons consists of various fields that are concentrated in a very small space without definite boundaries. The 'radius' which comes in at a maximum of 10-22 m, is completely arbitrary, and was chosen to represent a distance from the centre of the fields to where the electric field decayed sufficiently. Ergo, the electron itself is a tiny version of the 'electron vapour'. The only difference between the bound and unbound electron would be the diffusivity gradient and shape depending on which orbital was occupied.
P.S. From the article, 'point particle' is just a heuristic to describe the relative dimensions of a partcile. Plasmic Physics (talk) 19:36, 9 February 2013 (UTC)[reply]

Concluding remarks? Plasmic Physics (talk) 02:49, 11 February 2013 (UTC)[reply]

lemme toss the uncertainty principle into the mix; the uncertainty referred to is split between the position and momentum of the electron (in this instance). So, if you know the position of the electron to a high degree of accuracy, you have no idea what the momentum might be (i.e., is it moving or not?); alternately, if you know the velocity of the electron, you can't know what the position is, i.e. is the position changing. So, "is the electron truely whizzing about the atom in a random way, or is it everywhere around the atom at once"? You're asking for both position and momentum simultaneously, which makes this a question which cannot be answered, or more accurately, a question which is basically meaningless at this scale. Gzuckier (talk) 04:59, 11 February 2013 (UTC)[reply]

Covering bare floorboards

Okay, so basically today I put the wrong kind of soap in the dishwasher (liquid instead of powder :( ) and apparently the dishwasher didn't like that. I found this out as an hour after I started the load, I got up to check it, and found the kitchen floor F**KING FLOODED IN LATHER AND SUDS! Needless to say, i put away the dishes and cleaned up the mess, and the kitchen floor was so clean afterwards I could see my reflection in it. It was then I decided that as long as I was working around the bottom of the dishwasher, I might as well nail down the curled up linoleum around the edges of the floor. So I got out a box of 1&1/4" nails and a hammer and started pounding away.
However, some portions of the floor around the dishwasher and sink were so curled, distorted, and cracked that I couldn't nail them down, I had to cut off and throw away the ends before doing so. This led to a small, but long area of bare floorboard under the sink and dishwasher. This is a problem because:

  • The floor is particle board, so can cause splinters in the feet if stepped on wrong.
  • In addition to the water damage that the floor has doubtless suffered through the years already, it is now completely bare and any liquid spilled on the floor in that area could damage it.


So anyway, my problem is I can't decide which substance would be best for a sort of footstop under the counter. What I want to do is take one of those inclined thresholds like the sort under a front door to a house and nail it down there. I need a footstop which can prevent water damage and accidental kicking of the baseboards, hold the linoleum down indefinitely without cracking it, and last for up to five years.
My options are:

  • Metal
  • Rubber
  • Plastic
  • Treated Wood
  • More linoleum flooring
  • Tile


So, which would be best? (PS I filed this under science because I thought this was an engineering topic) --Free Wales Now! what did I screw up?  00:56, 9 February 2013 (UTC)[reply]

Free Wales now? Is Jimbo in jail? Oh, wait... :)
Will a high dam sill[1][2][3] do what you want to do? They come in long versions for use with garage doors. --Guy Macon (talk) 02:30, 9 February 2013 (UTC)[reply]
A bare margin with some sort of sill is not going to do you well if you have another flood. You're probably better of with a new floor that sits flush against the counter although I am sure you don't want to hear that. I won't laugh at your situation having gone through it myself and had a friend go through it last month. μηδείς (talk) 02:49, 9 February 2013 (UTC)[reply]
Thank you, that high dam sill thing does sound exactly what i'm looking for. Since my dishwasher seems to have flooded from under the door rather than the baseplate, needing a new floor shouldn't be a problem. Pretty much all I needed was a durable "ramp" to allow water to drain out onto the rest of the floor and off the baseboards while looking decent. Heading to Home Depot tomorrow... --Free Wales Now! what did I screw up?  03:41, 9 February 2013 (UTC)[reply]
It is altogether possible that some hose behind will develop a small leak at some time and what you are doing would hide the damage and make a pool to soak into the floor. Just fix the floor properly and cover the edge between the wall and the floor behind to stop any small leak into the crack. Dmcq (talk) 11:38, 9 February 2013 (UTC)[reply]
Note that the Refdesk does not give advice, accepts no liability if black mold spreads throughout your house causing serious and permanent illness that can be palliated only by expensive visits with a mind-body therapist, and we are not licensed interior designers[4] and therefore are not allowed to give interior design advice in Florida. Except in residences. Or is it Virginia now? Heck, it's a racket every bit as legitimate as the "medical advice" thing they're always on about here. Wnt (talk) 17:35, 9 February 2013 (UTC)[reply]

why was rutherford model of an atom discarded ?

Rutherford model was discarded because according to that model electrons will radiate energy as they move around the nucleus. In Bohr model, electrons revolve around the nucleus without radiating any energy. How is this possible ? The case (radiating energy), which was applicable for Rutherford model, was not applicable for Bohr model, why. --Concepts of Physics (talk) 03:05, 9 February 2013 (UTC)[reply]

For convenience, see Rutherford model and Bohr model. Basically, the Bohr model was just more specific than the Rutherford model. Rather than predicting the electrons existed in a cloud, possibly orbiting the nucleus, Bohr predicted very specific orbits. His model never explained how stably orbiting electrons fail to radiate energy, which was part of why physicists knew there was still a deficit in their understanding of the electron. Someguy1221 (talk) 03:18, 9 February 2013 (UTC)[reply]
An easy way to think of the historical chronology of these models is this:
  1. Thomson: "atoms are not indivisible, but are made up of negative subatomic particles in complicated orbits, all within a diffuse positive field"
  2. Rutherford: "actually, atoms are a dense positive nucleus around which tiny negatively subatomic electrons orbit in a classical way, but I don't know why they don't lose energy"
  3. Bohr: "ah, they don't lose energy because they're quantum, not classical, and in fact the way they gain/lose energy has to do with discrete stable states. But I don't know why that is other than to say that the quantum world is strange."
  4. De Broglie: "ah, the reason they are distinct stable states is because there are only so many stable wave functions, and the electrons are basically waves"
It's not so much that Rutherford was "discarded" so much as "built-upon." Ditto with Bohr's, which wasn't the last step there either. (Nor was de Broglie's, of course.) Obviously I'm simplifying the technical stuff here but such is how it goes with simplifications! --Mr.98 (talk) 18:05, 9 February 2013 (UTC)[reply]
In fact, the "plum pudding" model works pretty well for many applications in classical mechanics. You only need to worry about the breakdown of this model when you perform experiments with resolution better than the scale length of the atom. That means you're sending down photons of particular wavelengths (so you need the mechanics of Compton scattering; or you're analyzing emission spectra of atoms with high resolution. If you apply classical theory to the plum pudding model, and your scale length is always sufficiently larger than the atom, you will never find any conundrums! In formulations of quantum mechanics, this is an important and often-underemphasized empirical fact. Mathematically, this corresponds to a boundary condition, or a limit case, of any equation proposed to describe a quantum mechanical system: when extrapolated to large numbers, or large sizes, the formulation must remain consistent with what we observe! It is my opinion that most of the "conundrums" people encounter when they try to study quantum mechanics would be straightforwardly resolved if people would review the mathematical conceptual leap in the limit formulation of these problems. Nimur (talk) 20:01, 9 February 2013 (UTC)[reply]
See this is related to your question. Sunny Singh 09:03, 10 February 2013 (UTC) — Preceding unsigned comment added by Sunnysinghthebaba (talkcontribs)
Quantum theory revolutionized physics at the beginning of the 20th century, when Max Planck and Albert Einstein postulated that light energy is emitted or absorbed in discrete amounts known as quanta (singular, quantum). In 1913, Niels Bohr incorporated this idea into his Bohr model of the atom, in which an electron could only orbit the nucleus in particular circular orbits with fixed angular momentum and energy, its distance from the nucleus (i.e., their radii) being proportional to its energy. Under this model an electron could not spiral into the nucleus because it could not lose energy in a continuous manner; instead, it could only make instantaneous "quantum leaps" between the fixed energy levels. When this occurred, light was emitted or absorbed at a frequency proportional to the change in energy (hence the absorption and emission of light in discrete spectra).
Bohr's model was not perfect. It could only predict the spectral lines of hydrogen; it couldn't predict those of multielectron atoms. Worse still, as spectrographic technology improved, additional spectral lines in hydrogen were observed which Bohr's model couldn't explain. In 1916, Arnold Sommerfeld added elliptical orbits to the Bohr model to explain the extra emission lines, but this made the model very difficult to use, and it still couldn't explain more complex atoms. Want to be Einstein (talk) 10:09, 10 February 2013 (UTC)[reply]
This reminds me of my favorite piece of dumb management/marketing speak, where something or other is described as a "quantum leap" in blahblahblah. Someday I'll stick up my hand and say "You mean it's the smallest possible change?" Gzuckier (talk) 05:05, 11 February 2013 (UTC)[reply]

Why is the GAU-8 effective?

Why is the GAU-8 Avenger effective at destroying tanks when its caliber is only 30 mm and modern MBTs are designed to reliably withstand hits from 120 mm guns? Whoop whoop pull up Bitching Betty | Averted crashes 05:30, 9 February 2013 (UTC)[reply]

Diameter is not a determinant for destructiveness against armor; note that sabot anti-tank rounds have a "shoe" (fr. sabot) that separates from the round to release a kinetic energy projectile which is smaller than the diameter of the barrel from which it is fired.   From the article:   For reasons why a smaller diameter projectile can be desirable, see external ballistics and terminal ballistics.   ~E:74.60.29.141 (talk) 06:02, 9 February 2013 (UTC)[reply]
It probably wouldn't be sufficiently effective against a modern MBT. This 1980 report about an attack by A-10s on Korean War era M47 Pattons says the 140 hits, only 17 penetrated the Patton's armour. A Patton has 100mm of steel armour; the later M1 Abrams variants have advanced reactive Chobham composite armour which give a protection equivalent to 600-1000mm of steel armour. That's why the A-10 carried higher performance weapons like AGM-65 Maverick as well, specifically for hard targets that the GAU-8 wouldn't kill. The GAU-8 is still useful against a wide variety of battlefield targets like AFVs, APCs, trucks, tankers, jeeps, artillery pieces, tractors, and self-propelled guns, and against unfortified structures. -- Finlay McWalterTalk 16:33, 9 February 2013 (UTC)[reply]
That said, it's not totally useless against an MBT; given the GAU's weight of fire, it's quite possible it would be able to detrack an MBT, rendering it ineffective and vulnerable. -- Finlay McWalterTalk 16:36, 9 February 2013 (UTC)[reply]
The M829's KE penetrator is only 20 mm in diameter. However the A-10's cannon is ineffective at killing modern MBTs. It can handle out of date T-62s and the likes, but not a post-1980 MBT. It can several damage them, destroying their tracks, vision equipment, even weapons and engine (weakly armored grille). A machine gun can be completely destroyed by a hit and I don't think a crew would risk firing a big gun that has been damaged by an autocannon, the shell might explode before leaving the barrel. All of this is repeairable though, for an assured detruction it must used its missiles. You must remember though that MBTs are only a small percentage of the vehicles in an army. Lightly armoured and soft skinned vehicles are the bulk, and the cannon can effectively destroy these without wasting a missile.--Whichwayto (talk) 17:11, 9 February 2013 (UTC)[reply]
For a kinetic kill system like a DU anti-armour round, the crucial factor is the projectile's kinetic energy. If I'm doing the maths right, the KE (at the muzzle) for a GAU-8 round is about 0.2 MJ; for an M829A3 from an Abrams its about 12 MJ. -- Finlay McWalterTalk 17:27, 9 February 2013 (UTC)[reply]
Actually, in the case of the Hog's GAU-8, the most crucial factor is hitting the top armor rather than the frontal armor -- a tank's armor is weakest on top. 24.23.196.85 (talk) 20:22, 9 February 2013 (UTC)[reply]
ineffective at killing modern MBTs is incompatible with what follows that claim. A blind, motionless, gunless MTB might as well be dead. It's certainly a mission-kill, and it's an easy target for infantry with any kind of anti-armour. --Stephan Schulz (talk) 21:18, 9 February 2013 (UTC)[reply]
But, as he said, almost all of that damage is repairable by the tank crew. Just bolt on the spare machine guns, realign the main gun, fix the tracks, and you're ready to go. Even if some of it isn't repairable, the tank can still be useful; for instance, if the periscope is wrecked, the commander can still effectively direct fire by standing up in the cupola (good tank commanders do this anyways), and even if the tank is rendered completely motionless, it is still quite useful as a fully traversable, armored artillery piece. Whoop whoop pull up Bitching Betty | Averted crashes 17:16, 10 February 2013 (UTC)[reply]
Although modern armies are very efficient at recovering and repairing damaged tanks, as long as you're advancing and not retreating. Another point is that modern armies have a great many more armoured vehicles that are NOT main battle tanks; armoured personnel carriers, armoured reconnaissance vehicles, self propelled guns, armoured command vehicles, and so on; it is these that the "Avenger" is really intended to counter. Alansplodge (talk) 13:07, 10 February 2013 (UTC)[reply]

do we know what Mars sounds like?

did Curiosity or any other mars probe have a few grams to spare out of their multiton payload on a microphone, so we could hear what Mars sounds like? 178.48.114.143 (talk) 06:46, 9 February 2013 (UTC)[reply]

One noise emanating from this phenomenon sounded something like "Illudium Q-36 Explosive Space Modulator." ←Baseball Bugs What's up, Doc? carrots07:01, 9 February 2013 (UTC)[reply]

http://www.youtube.com/watch?v=DJ3A4XOWSFE

http://curiositywatch.com/sounds-from-mars-why-curiosity-has-no-microphone/

Considering the rover is two tons, the mic would have added precisely two grams and a single analog input, and was not mission-critical, would not need to be pointed anywhere and can literally be in any crevice, does not need to even be automatically collected or read or affect any other mission requirements, can break or fail to function without consequences, yet if it happened to still work, would bring huge outsider interest in mars, I hope you will agree that I am simply smarter than NASA. That video, until the rickroll, was absolutely FASCINATING. I was like, wow, wow, wow. Two grams. Fuck you, stilted engineers. You didn't even try! 178.48.114.143 (talk) 09:01, 9 February 2013 (UTC)[reply]
I'm sorry to disappoint your hope, but I don't agree. I suspect that the predominant sounds would be those of the motion of the rover. Are those of interest? I'd be surprised if a Mars microphone picked up any interesting sounds. (dinosaurs? little green men chatting? ) Dbfirs 09:30, 9 February 2013 (UTC)[reply]
It's impossible to predict what scientific discoveries will be made in advance, or else they wouldn't be discoveries. For example, the LHC was predicted to discover the Higgs boson, a particle that's been part of the Standard Model for decades, and that nobody seriously doubted the existence of. If that's all it discovers, the LHC is a huge waste of money, but physicists are hoping for something unexpected that will revolutionize physics. What will that be? Well, if anybody knew, it wouldn't be a discovery. --140.180.247.198 (talk) 17:41, 9 February 2013 (UTC)[reply]
Mars... I can't believe I'm back on Mars. Three times before, this place almost killed me. I swore I'd never give it another chance to finish the job. Humns got no business being here. No business at all.Michael Garibaldi. How different is that from Saigon...shit? --Trovatore (talk) 09:36, 9 February 2013 (UTC)[reply]
Yes, the sounds of the motion of the rover would have been very interesting. Out of curiosity, why do you think we wouldn't have heard wind, as in the video? Or explosions in the distance from volcanos nearby, which the rover cannot see visually? Acoustics is so so cheap. If nothing else, the sound of the gravel/sand/whatever under the rover's feet... This article says it is "probably still volcanically active today": http://en.wikipedia.org/wiki/Volcanology_of_Mars - so, out of curiosity, if the rover happened land within earshot of a volcano, you don't think having a mic onboard would be something anyone would be interested in hearing? No mic is simply irresponsible. It's not like I'm asking you to include speakers, so we can blast some music and hear its echoes if we are near a wall. I'm asking for a two gram mic. There is no excuse. Aren't there dust storms, too? You don't want to know what they sound like? No imagination around here. 178.48.114.143 (talk) 10:14, 9 February 2013 (UTC)[reply]
“Curiouser and curiouser!” Cried Alice: I find usage of the cliché out of curiosity curious interesting in a thread discussing NASA's Curiosity rover --Senra (talk) 22:14, 9 February 2013 (UTC) [reply]
If we prorate the ENTIRE budget of $2.5 billion just into the 2000 pound payload - which is a huge overstatement, imagine if a scenario where the actual final payload is just 100 grams. Making it 101 grams by adding a gram to what we consider, wouldn't have cost a further marginal $25 million or increased the budget by a full percentage point! Most of the budget is in developing the rover, delivery system, etc. So this is a huge, huge, huge, huge, HUGE overestimation. Still, by this HUGE overestimation, another 2 grams added to the payload would have increased the size of the payload by 1/500000th, or the prorated budget from $2.5 billion by another $5,000. A handful of people, hell, a single individual, would have gladly paid that amount to hear Mars. If the only place a single recording ever appeared was youtube, the advertising revenue would be more than $5,000 from it. Think about it!!! There is just no excuse, except ignorance and lack of creativity. 178.48.114.143 (talk) 10:30, 9 February 2013 (UTC)[reply]
"ignorance and lack of creativity", eh? And yet ironically they're the ones driving a robot on Mars and you're the one wasting your time ranting on an internet forum which isn't actually a forum at all.Dncsky (talk) 13:36, 9 February 2013 (UTC)[reply]
I wouldn't be so quick to discount the OP's opinions, though some of them could have been expressed in a more rational fashion. I, for one, wouldn't mind listening to dust storm or similar event taking place on Mars (imagine all the relaxation CDs off-brand record labels could sell in five-and-dime shops!), and I'm certain the cost would have been neglibible compared to the other components of the rover. Let's also not forget that technical know-how does not necessarily correlate with creativity or a propensity for brilliant ideas (for example, this is a product that smart people allow to exist). Evanh2008 (talk|contribs) 13:58, 9 February 2013 (UTC)[reply]
This question has been asked before, here. Here's one of the responses:
"Plenty of people have pushed for including a microphone on a Mars mission, most notably the Planetary Society, whose microphone actually flew on the failed Mars Polar Lander. The Phoenix lander also included a microphone, but only because its descent imager (MARDI) happened to have a microphone on its circuit board. MARDI was turned off because it had a risk of interfering with IMU measurements during landing, which could have been fatal to the spacecraft. According to an interview I heard with a Planetary Society member (Emily Lakdawalla), the Society lobbied to have their microphone included on Curiosity, but it was rejected because the rover was already complex enough (she didn't elaborate on this, and I haven't been able to find a more detailed explanation). Personally I think that's a strange reason, because a microphone is one of the smallest and simplest electronic devices possible (just look at your cellphone to see how small it could be). Having one would be great for PR, and NASA only gets funding if the public is excited and inspired by its missions."
Here's an answer from the Curiosity team themselves: [5]. The second part is interesting:
"Here's a little more info on the Phoenix microphone. It was essentially a hitch-hiker. It was built into another instrument taken off the shelf for the the lander, but it was never intended for the mission. There was no science team or budget connected to it. Since it was not intended for use it was never tested before launch and never entered into the power budget for the lander. Only after Phoenix successfully completed it's mission, 5 months after landing in the polar region, was the mission somewhat willing to test it. They couldn't do it earlier because they couldn't risk the prime goals of the mission if anything went wrong. The project manager was fairly certain it wouldn't work and was against trying it because he didn't want to raise expectations. His mind changed when we got a tweet to the @MarsPhoenix account from a man who said he was blind and how much he wished he could hear Mars because he couldn't see the pictures. A couple days later, they sent the signal to Phoenix to turn it on but we got.. well.. nothing. Empty files. If we had received anything, it would have been released. The team figured the mic was frozen solid and decided to give it a second try by leaving it on longer to warm up. Unfortunately, the Phoenix mission lost its last bit of power (as expected) before it got the second instruction." --140.180.247.198 (talk) 17:41, 9 February 2013 (UTC)[reply]


Let's rephrase the answer another way. Spaceship engineers are not stupid. They know what their instruments measure. Now, let's review this carefully, because it is not clear whether the OP knows what a microphone measures.
"Sound, obviously!" says the enthusiastic OP! "I want to record sounds! Put a microphone on the spaceship!"
Now, as the reference desk regulars who follow Science know, we've had a recent spate of questions asking about minimum densities that sound can propagate in. And, as we have repeatedly answered, there is no well-defined answer to that question. So, "is there sound on the Moon? Is there sound on Mars?" While our knee-jerk response is "of course not, these places are nearly total vacuums, and sound cannot propagate!" ... if we stop to really really carefully answer the question, we find that we must refine this to say, "...well, actually, the amplitude of acoustic waves in very-very-sparse atomspheres are very tiny, and the frequencies are very low."
This brings us back to my first point: spaceship engineers are very smart. So while you think "a microphone measures sound" and cuss out the engineers, let me defend their line of reasoning. A microphone doesn't measure sound at all. Most modern "2-gram" microphones are piezoelectric crystal attached to high gain differential amplifiers, and then digitally sampled. They measure a voltage, proportional to the mechanical stress on a piezoelectric crystal. When a sound is audible to a human, that sound corresponds to a pressure wave propagating in air, which will exert a mechanical stress on a collector plate and deform the piezo, inducing a voltage. That voltage will be amplified and sampled.
We can calculate exactly what voltage will be produced for any given deflection.
We know the density of the Mars atmosphere, and we know its pressure, and we know how much force a molecule can exert on a piezoacoustic coupling... and we know the electronic noise floor of the finest differential amplifier circuit this side of the Jovian moons... and we know the realistic sampling rate and quantization error for digitizers, because we studied elementary digital design theory... and the net result is: the signal will be all zeros.
If we know (from other experiments, and from application of the laws of fundamental physics, that it is not physically possible to measure signal, given the sparse atmosphere of Mars, we don't waste time putting a microphone on the spaceship.
Instead, we go back to basics: what are we trying to measure? Vibrations? Particle flux? We have instruments on Mars Science Laboratory to do that. MEDLI, RAD, and others.
So, the real point here is, the OP is making an unreasonable demand to place an instrument known not to work on a billion-dollar spacecraft. When the OP finances, engineers, and executes his/her own billion dollar spacecraft, they may place any instrument they like onboard ...and if that is your objective, start by reading my post on engineering tradeoffs in spaceship design, with links to a complete lecture-series... : but until then, accept at face-value this simple fact: somebody very smart thought about your idea and dismissed it for very good reasons, long before you even knew they were planning a Mars rover. Nimur (talk) 20:25, 9 February 2013 (UTC)[reply]
OMG, did you read anything that other people wrote before harping, falsely, about how it's impossible to record sound on Mars? NASA's Mars Polar Lander had a microphone, by design, for the purpose of recording sound. Phoenix carried a microphone by "accident", and though it heard nothing, the team thought that was because it got frozen and gave it a second try. The Planetary Society has built a Mars microphone for inclusion on a spacecraft, and it flew on the Polar Lander. Your claim that it's physically impossible to hear sound on Mars is simply wrong. --140.180.247.198 (talk) 00:40, 10 February 2013 (UTC)[reply]
I agree. Unusually for Nimur, he was wrong in detail (eg most modern cheap microphones are electret, not piezo - piezo has been obsolete for 30 years) and wrong in overall thrust. While the low altitude atmosphere density on Mars is only about 1/50th of that on on Earth, this is still quite sufficient to carry sound - you'll just need a much bigger microphone diaphram area to get equivalent sensitivity, as was explained in another recent question on Ref Desk. Detectable frequencies can easily cover a good bit of the audio range. However I also agree that nothing much is lost by not having the microphone. What do you expect to hear? I should think nothing other than the mechanical noises of the spacecraft, and a bit of wind noise. We know what wind noise sounds like. It's not as though we seriously expect the sounds of alien construction work or something. A transducer to measure wind velocity would be MUCH more cost effective. Wickwack 121.215.84.193 (talk) 03:08, 10 February 2013 (UTC)[reply]
I harp about many things, and so I am occasionally wrong. In this instance, I regret that my post did not cover all possible types of acoustic transducers; and that I did not formally reference or calculate the exact data that we would record for a particular sound pressure level at Mars' surface. My hope is that the interested reader could follow my line of reasoning and trivially estimate these parameters themselves, using their favorite type of transducer, amplifier, sampler, compression algorithm, and so forth. Perhaps this was an instance where the exercise should not have been left to the reader. As a result of this omission, I did not personally calculate the expected signal level, and so I may have overemphasized my position. In review, I concur that it is not "physically impossible" to record acoustic signals on Mars; but I stand by my position: recording acoustic signals on Mars is "still not an effective use of engineering time or science budget." I apologize that my overemphatic post may have inadvertantly conveyed an incorrect conclusion.
But, regarding electrets and piezos: it is a fair point that most new computers use condenser microphones; but piezos are not obsolete; my electric guitar was built in 2004 and uses piezo devices similar to this, and they sound quite nice. I have strong reason to believe that they are linear to many hundred octaves above the human ear's capability to hear sound (and far beyond the ordinary frequency range of a condenser microphone). And, in other work, I've used piezo couples for lots of reasons, particularly when I need to put my microphone in a vacuum, or on a rocket, or in a radiation-hazard enviroment, or any other place I need to doubt the integrity of a condenser microphone. Nimur (talk) 06:21, 10 February 2013 (UTC)[reply]
We are off-topic now, but you'll find if you check, those mikes in laptop computers are electret, not condenser. However the operating principle in both is the same (charge pumping by varying the distance between electrodes), and some people are sloppy with their terminology. In any case neither electret nor condensor are piezo. For electric guitars, magnetic pickups are used almost exclusively. Only in acoustic guitars with non-steel strings are piezo pickups (such as the Palathetic )used, as obviously magnetic pickups will not work without steel strings. For studio work, a conventional mike will give a better sound for acoustic guitars, though of course just what the sound is, is a matter of choice between the musician and the producer. Your link returns a 404 error. The design of condenser mikes is a compromise (a rather good compromise compared to other types of mike, but still a compromise), explained extremely well in Harry F Olson's, Acoustical Engineering - a venerable but still excellent standard text on the subject. Condensor mikes are certainly NOT good for hundreds of octaves above human capability - that would be greater than 15 kHz x 2100 ie ~2 x 1033 Hz, well and truely beyond the range of theoretical physics! Visible light is only around 6 x 1014 Hz! You are being ridiculous. Condenser mikes, by the nature of how they operate, are not inherently linear but typically are good covering just the frequency and amplitude range of human hearing. Wickwack 120.145.68.197 (talk) 09:37, 10 February 2013 (UTC)[reply]
It would violate the law of physics, i.e. "in space nothing can hear you scream". Gzuckier (talk) 05:13, 11 February 2013 (UTC) [reply]

Why is my charger emitting high pitched sound?

I think it has something to do with the transformer, but I don't know why. — Preceding unsigned comment added by Inspector (talkcontribs) 07:04, 9 February 2013 (UTC)[reply]

It uses a Switched-mode power supply, and is probably a "cheapy" ~15 kHz - which is within normal human hearing range. ~E:74.60.29.141 (talk) 07:33, 9 February 2013 (UTC)[reply]
And I am guessing that the person who asked the question is either under 40 years old (if male) or under 60 years old (if female). Otherwise he/she would not be able to hear it. --Guy Macon (talk) 11:19, 9 February 2013 (UTC)[reply]
I guess the sound frequency is much lower. There is still one other strange thing when I studied further: when I connect it to my cell phone, the continuous buzzing stopped and changed to intermittent buzzing like heartbeat buzzing in the hospital.--Inspector (talk) 11:45, 9 February 2013 (UTC)[reply]
As far as I know this is an indication that it's either broken or or does not conform to the usual specifications required in industrialised countries (conformance is often claimed fraudulently). And that it's an incendiary risk. If it's expensive to replace, the device you are charging is not very valuable, and there is no reason to trust that a new charger will be better, then I personally would just continue to use it but would be careful never to keep it plugged in when I leave the house. Also, I would always think of it first when there are any problems with radio or tv reception, wlan etc. Hans Adler 11:52, 9 February 2013 (UTC)[reply]
Hans, I wouldn't go that far... it sounds like you're spreading a little FUD about a common, harmless annoyance. It's far more likely that the power-supply is functioning correctly, but its designers did not go through the painstaking analysis to guarantee that it never emits audible noise under any condition. If the device has a marking from UL - Underwriters Laboratory - or from the FCC, then its safety and emissions have been tested and verified, even if it makes an annoying noise. Outside the U.S., other agencies and industry groups are responsible for similar testing. This should not be construed to mean that any UL-tested product is always safe under any condition - you need to apply common sense and be aware of defects and damage. But, I am not aware of any requirement that audible noise in a switching power supply must be suppressed, and I don't understand why you think audible noise has any correlation to fire-hazard. Audible noise might be caused by any number of conditions, and most are not hazardous. In many scenarios, noise might indicate that the power supply is not operating at peak efficiency or that it's exerting unnecessary mechanical stress on some component, reducing its usable lifetime. But ... risk of fire? Surely if audible noise indicated fire-hazard, then every speaker cabinet would be a fire hazard during normal use? Nimur (talk) 19:46, 9 February 2013 (UTC)[reply]
A high-pitched buzzing noise might indicate sparking, but this is improbable. More likely this is the result of some transistor-induced harmonics causing the transformer core to intermittently vibrate and emit audible noise. 24.23.196.85 (talk) 20:27, 9 February 2013 (UTC)[reply]
I think Hans Adler is suggested that any device which emits noise must be a poorly made device which fails to comply with proper safety standards as present in most developed countries and any such markings suggesting otherwise are fraudulent so that the device must be a fire risk; not that noise itself indicates a fire risk. I agree this is going way too far. It may be slightly more likely a improperly designed or made power supply which violates safety standards makes audible noise, but there are a large number of perfectly safe devices which comply with whatever local standards which also make noise. And there will be plenty of unsafe devices which don't have noise.Nil Einne (talk) 21:18, 9 February 2013 (UTC)[reply]
Modern chargers are almost certainly switched-mode power supplies (SMPS). Our own article suggests such supplies emit acoustic noise (almost a tautology?) suggesting that such noise is "[u]sually inaudible to most humans, unless they ... are malfunctioning, ...". I think that means unless the SMPS is malfunctioning, not the human. Also "[t]he operating frequency of an unloaded SMPS is sometimes in the audible human range, and may sound subjectively quite loud for people who have hyperacusis in the relevant frequency range". QED? --Senra (talk) 21:55, 9 February 2013 (UTC)[reply]
Sincere apologies to 74.60.29.141 (talk · contribs) who mentioned SMPS above. Didn't see that. Sorry. --Senra (talk) 22:02, 9 February 2013 (UTC)[reply]
I don't think I am alarmist, just a bit careful after I have read several reports like this one. Hans Adler 22:20, 9 February 2013 (UTC)[reply]
I have a bit of experience in this area, having designed everything from components for NASA manned space flight to toys for Mattel. Warnings are good, but they have to be accurate. Otherwise you are chicken little or the boy who cried wolf. The fact of the matter is that there is no known correlation between SPS acoustic noise and safety. You are warning people about the wrong things. --Guy Macon (talk) 23:26, 9 February 2013 (UTC)[reply]
I would also note that the example you gave (the counterfeit power supply with no components that fed the 12V straight through instead of the proper 5v) is completely silent and very unlikely to malfunction any more than it is already doing... --Guy Macon (talk) 00:56, 11 February 2013 (UTC)[reply]


It could be a phasor on overload, so toss it down the nearest disposal chute right away ! :-) StuRat (talk) 23:29, 9 February 2013 (UTC) [reply]
What! The resulting explosion might kill the princess! --Senra (talk) 17:22, 10 February 2013 (UTC) [reply]
Back to the topic, is it functioning right for the charger to change to periodical beeping when connected?--Inspector (talk) 02:02, 10 February 2013 (UTC)[reply]
It depends what you mean by beeping. If it is charging something that has charge control circuitry, the load might be switching on and off, but I wouldn't expect this to be a fast "beep". A slow cycling with the high frequency periodically going on and off might just be caused by the noise disappearing when current is drawn. This could well be normal operation because it is not unusual for the high frequency switching to induce a tiny vibration in laminations of the core, especially under no-current conditions. Technically, this would be regarded as a "fault" because energy is being wasted, but it is a very minor fault. If the charger is getting very hot, then there might be a more serious fault, but just warm is normal. Dbfirs 12:56, 10 February 2013 (UTC)[reply]
You have not told us the age and model of your cell-phone, nor whether the charger is the original or a low-cost replacement. If I had this problem, I would be searching the manufacturers forum to determine if others had reported similar problems. In your case, I would also eliminate other potential explanations. For example, responders above mention age and I noted hyperacusis as a possible cause. Have you asked non family members (in case hyperacusis is hereditary) if they can hear it too? --Senra (talk) 17:22, 10 February 2013 (UTC)[reply]

i think i've heard that Leptin inhibits Gerlin. is that true?

thanks. 79.179.134.180 (talk) 16:32, 9 February 2013 (UTC)[reply]

If you only think you've heard something, it is unlikely to be true. Anyway, what is gerlin? Is it like trollin?--Shantavira|feed me 16:35, 9 February 2013 (UTC)[reply]
In a quick scan of our articles I didn't see any discussion of that question. There is some evidence that leptin inhibits ghrelin, for example PMID 15867335, but the data seem to be limited. Looie496 (talk) 16:59, 9 February 2013 (UTC)[reply]
...which is why I said what I said. -- Scray (talk) 02:55, 10 February 2013 (UTC)[reply]

in simple words

how do i culculate my bmi index?, please give me a verbal explanation. thanks 79.179.134.180 (talk) 16:32, 9 February 2013 (UTC)[reply]

Please see Body mass index or http://simple.wikipedia.org/wiki/Body_mass_index --Shantavira|feed me 16:38, 9 February 2013 (UTC)[reply]
Or go to http://nhlbisupport.com/bmi/ and the US Department of Health will calculate it for you. --Guy Macon (talk) 16:46, 9 February 2013 (UTC)[reply]
Measure your height (in meters) and your weight (in kilograms), and plug them into this formula:
.
That's all there is to it. Looie496 (talk) 16:48, 9 February 2013 (UTC)[reply]
Weigh yourself, in kilograms. Measure your height, in metres. Square your height (that is, multiply your height by itself). So, if you are 1.5m tall, you do 1.5 times 1.5 (1.5 x 1.5). Make a note of your answer. Take your weight, and divide it by your previous answer. This number is your BMI. 86.163.209.18 (talk) 12:34, 10 February 2013 (UTC)[reply]
Or if you prefer pounds and inches, use
. Duoduoduo (talk) 13:28, 10 February 2013 (UTC)[reply]

Graphite

I think I have found a new way of synthesising graphite. In order to be certain that it is graphite, I need to know whether the route of production is a possibility.

Crystallisation of graphite from solution. I added organic material to 38% sulfuric acid, and boiled until the material was fully carbonised. The resulting product was a highly acidic, opaque, black sol (colloid)(?). Upon, cooling and standing for several hours, large, insoluble, opaque, black crystals have formed. I could send photos of the mystery crystals. Plasmic Physics (talk) 22:28, 9 February 2013 (UTC)[reply]

I suggest you try writing on paper with it, then using an eraser on it. If it acts just like graphite in both respects, it probably is graphite (lead behaves similarly, but I can't see how you could have produced lead without having put lead in). Of course, graphite is quite cheap, so you probably aren't going to make it any cheaper than the current methods. StuRat (talk) 22:31, 9 February 2013 (UTC)[reply]
It quite possibly is graphite. Adding oleum to sugar will produce a ball of carbon, that is larger that the reactants you started with. CS Miller (talk) 22:58, 9 February 2013 (UTC)[reply]
(edit conflict) The black stuff you made is the product of the dehydration reaction between the sulfuric acid and the organic material (a typical reaction for sulfuric acid, I should add), and as such, is essentially pure carbon. However, I should caution you that it might still contain free acid, which would tend to damage the paper. 24.23.196.85 (talk) 23:01, 9 February 2013 (UTC)[reply]
I'll wash it several times in water. Plasmic Physics (talk) 23:14, 9 February 2013 (UTC)[reply]
Considering that the sulfuric acid predominantly acts as a catalyst, and the reactant is essentially free, I fail to see how it would not be cheaper? Plasmic Physics (talk) 23:14, 9 February 2013 (UTC)[reply]
Well, the energy needed to boil it until carbonized isn't free. StuRat (talk) 23:31, 9 February 2013 (UTC)[reply]
Put it in perspective with the cost of mining, or generating thousands of degree temperatures for synthesis. Plasmic Physics (talk) 00:00, 10 February 2013 (UTC)[reply]
Should we ask the name of your unfortunate victim which you describe merely as "organic material" ? :-) StuRat (talk) 23:34, 9 February 2013 (UTC) [reply]
Don't ask, Stu. We do not out people here (unless it is public knowledge anyway) --Senra (talk) 23:52, 9 February 2013 (UTC) [reply]
I call him Cooking Oil. Plasmic Physics (talk) 00:00, 10 February 2013 (UTC)[reply]
Or perhaps "Cooking Earl" ? StuRat (talk) 03:56, 10 February 2013 (UTC) [reply]
Is your organic material free? (don't forget transportation costs!) Is the energy used to boil the acid free? Is the amortized cost of the equipment free? Is the hazardous waste disposal that your washing step requires free? Labor? Renting a building? Remember, you are competing with people who are digging graphite out of the ground in China. --Guy Macon (talk) 23:39, 9 February 2013 (UTC)[reply]
According to the charcoal article, creating charcoal by the dehydration of sugar by sulfuric acid will produce the purest charcoal. CS Miller (talk) 23:45, 9 February 2013 (UTC)[reply]
Pond scum should work, 'organic material' is a pretty wide menu. As for waste desposal, such a plant could be adjacent to a sulfuric acid plant/phosphate fertilizer pant. The hardous waste could be recyled thus. Plasmic Physics (talk) 00:00, 10 February 2013 (UTC)[reply]
I've known people who work in the industry, so I feel safe saying that sulfuric acid production is nasty business. It produces gobs of really nasty waste which is a bitch to take care of, and we're not likely to be able to produce it in volumes necessary to produce enough fuel given that it is already, by far, the most produced chemical in the world. In order to make a viable source of carbon as a fuel to replace mined coal, you'd need more sulfuric acid than is availible. In 2011, the world produced 7,695.4 million tonnes of coal. That's 7,695,400,000,000 kilograms of coal. To produce a similar amount of carbon from random carbon sources, you'd need at least a 1:1 stoichiometric amount of sulfuric acid; probably a lot more. That'd be 7,695,400,000,000 *98/12 = 62,846,000,000,000 kilograms of sulfuric acid. The world produces about 180,000,000,000 kilograms per year, so even if you converted the entire world production of sulfuric acid over to your plan, you'd create enough pure carbon to replace 0.2% of the world's coal output. So, you're "cost of mining" issue is irrelevent. You couldn't possibly make enough carbon to make a difference in mining operations. --Jayron32 04:57, 10 February 2013 (UTC)[reply]
Just a note: this subdiscussion is about synthetic graphite, not coal. Plasmic Physics (talk) 05:47, 10 February 2013 (UTC)[reply]
You brought up the coal mining, not me. To what end are you proposing to use the graphite? Is there a shortage of graphite or is the economics of the current graphite market in sore need of you scaling up a middle-school classroom demonstration into an industrial process? --Jayron32 06:03, 10 February 2013 (UTC)[reply]
That was the first time I "coal" escaped my mouth, in a manner of speaking. Well, I think so. Plasmic Physics (talk) 08:46, 10 February 2013 (UTC)[reply]
Well, even if you were wanting to replace coal, it's not entirely out of the question. Firstly, you don't need to replace all coal, coking coal is the most valuable coal commodity, so extra costs of production are more easily absorbed there. Also, if you use a renewable source of organic material, then your material is carbon neutral (ignoring the energy input required which may or may nor be fossil fuel based), which gives it an advantage in a market with carbon trading or carbon taxes. Also, Jayron, who is clearly a very good physicist, is no economist. If there was a massive extra demand for sulfuric acid, there would be a massive incentive to increase supply. Unless we've already allocated every applicable resource to producing sulfuric acid, there's always more production possible. 202.155.85.18 (talk) 01:41, 11 February 2013 (UTC)[reply]
Unless we're already at or near capacity for producing sulfuric acid, then all we're likely to do is drive up the price or divert supplies away from other uses. --Jayron32 03:00, 11 February 2013 (UTC)[reply]
The price will be driven up by the increased demand. That is a given. You can only divert from other uses by offering a higher price. If we're at capacity for H2SO4 production, then we can only divert from other uses. 202.155.85.18 (talk) 03:42, 11 February 2013 (UTC)[reply]
(edit conflict) And what of the capital investment required for new sulfuric acid plants, which would run into the hundreds of megabucks? (Not to mention the other barriers to entry, like environmental regulations.) Also, graphite CANNOT replace coking coal -- coke must be not only chemically pure, but hard as well (because it must not crumble to dust when dropped a hundred feet or more down a blast furnace along with a tonload of iron ore and limestone), whereas graphite (no matter whether it's natural graphite, Acheson graphite or Plasmic graphite) is soft and easily abraded. 24.23.196.85 (talk) 03:06, 11 February 2013 (UTC)[reply]
Plasmic's exact phrase was "cost of mining" -- so he said "mining", not "coal". Graphite is a different material than coal, with different applications, but it's found naturally and mined in a similar manner to coal. It can also be produced synthetically by carbonization of coal tar pitch, which is more economical than Plasmic's method of dehydrating sugar or cooking oil with sulfuric acid (coal tar pitch being a major industrial waste from the steelmaking industry, and the raw materials for the Plasmic Process being more expensive, along with the prohibitive cost of hazardous waste disposal in the latter process). 24.23.196.85 (talk) 01:49, 11 February 2013 (UTC)[reply]
Raw material, as I've said, includes most organic material, even waste. Using cooking oil, or sugar would not be wise choices in this regard. Oil was merely used as a proof of concept. And sulfuric acid is not a main reactant, but a catalyst, it is ordinairily only slowly consumed. Plasmic Physics (talk) 02:47, 11 February 2013 (UTC)[reply]
It is gradually diluted and contaminated, and eventually must be disposed of as hazardous waste, every drop of it. Also, when the graphite is washed to remove the free acid (which would otherwise corrode anything that comes into contact with Plasmic graphite), the wastewater also has to be disposed of as hazardous waste. Sorry, the Acheson process is still much more economical than what you propose, even with the high energy costs these days. 24.23.196.85 (talk) 02:53, 11 February 2013 (UTC)[reply]
What are all of these supposed toxic byproducts? The sulpfuric acid itself ig gradually diluted with water, so you have dilute sulphuric acid which is great for making...concentrated sulfuric acid. Even if you did want to dispose of it, just add lime to balance the pH and discharge. 202.155.85.18 (talk) 03:39, 11 February 2013 (UTC)[reply]
This "dilute sulfuric acid" is actually UNFIT for making concentrated sulfuric acid, due to extensive contamination with organics and metals and whatnot. Purifying it before reconcentrating would be even MORE expensive than its disposal. 24.23.196.85 (talk) 04:15, 11 February 2013 (UTC)[reply]
It's not going to have "extensive contamination with organics". How do you remove organics from things? You percolate it through finely divided carbon. What was just in the acid? Oh yeah, finely divided carbon. And disposing of it is not expensive anyway. And there's no real need for the conc H2SO4 to be ultra pure anyway. The whole "recycling sulfuric acid" is not up for debate since it's a stable of industry, as in sulfuric acid recovery plants. 202.155.85.18 (talk) 04:35, 11 February 2013 (UTC)[reply]
I think you're thinking of activated carbon, not finely divided carbon. Plasmic Physics (talk) 18:04, 11 February 2013 (UTC)[reply]
Evaporate the liquid, purify and recycle the sulfurous oxides back into sulfuric acid. Using various separation techniques, the only waste should be in the form of a solid residue (metal oxides and sulfates). Plasmic Physics (talk) 03:41, 11 February 2013 (UTC)[reply]
And the energy needed for the purification and evaporation would cost how much? 24.23.196.85 (talk) 04:16, 11 February 2013 (UTC)[reply]
Without reviewing the chemistry, I think this sounds like a plausible notion to get coal out of biodiesel, but don't people actually want liquid fuels out of coal? Wnt (talk) 16:08, 11 February 2013 (UTC)[reply]
Not coal, but graphite. Plasmic Physics (talk) 18:04, 11 February 2013 (UTC)[reply]

I used piranha solution today. Heating it is a bad idea, unless you want a steaming hot acid volcano. Plasmic Physics (talk) 01:16, 12 February 2013 (UTC)[reply]

(un-indent) Plasmic and 202 IP, you know your chemistry very well, but it's clear you're no engineers. I recommend you look in the Kirk-Othmer Encyclopedia of Chemical Technology and see for yourselves just how involved and energy-intensive the process for re-concentrating waste sulfuric acid is in real life. 24.23.196.85 (talk) 03:11, 13 February 2013 (UTC)[reply]


February 10

Annihilation

Would an electron and positron annihilating be considered two particles occupying the same space?GeeBIGS (talk) 02:21, 10 February 2013 (UTC)[reply]

Not after they were annihilated (which means they would have ceased to exist). -- Jack of Oz [Talk] 03:44, 10 February 2013 (UTC)[reply]

Well how about before or during then? Sounds like the same exact place to me....NOWHERE.GeeBIGS (talk) 06:20, 11 February 2013 (UTC)[reply]

No, both electrons and positrons are fermions which, by the Pauli exclusion principle cannot occupy the same quantum state (the concept of "same space" at this scale is somewhat meaningless; we don't speak in terms of location, since these particles are not localizable, but instead we speak of particles occupying a certain "quantum state", which is functionally equivalent to "location" for the purpose of discussions like this). However, bosons can occupy the same quantum state as each other; this leads to materials like Bose–Einstein condensates. --Jayron32 04:40, 10 February 2013 (UTC)[reply]
To my understanding, the Pauli exclusion principle only applies to the full quantum state of fermions. Any difference in quantum numbers will allow cohabitation - which is why you can get two electrons in one atomic orbital. They share quantum states except for one is "spin up" and the other is "spin down". I'm not familiar enough with QED to know the answer, but wouldn't the difference in charge on electrons and positrons be sufficient to introduce them into different quantum states, even if everything else about them was identical? If so, then the Pauli exclusion principle would say nothing about them regarding being able to occupy the same space or not. -- 205.175.124.30 (talk) 01:53, 11 February 2013 (UTC)[reply]
Well, that's why I hedged over the use of the term "same space". What happens is their probability space overlaps as they get closer, and eventually the likelihood of annihilation becomes great enough to cause it to happen, and it does. They can do this without the "points" occupying the "same space", or even having the same quantum state. --Jayron32 04:09, 11 February 2013 (UTC)[reply]

So how close do they get to each other before becoming empty space?68.36.148.100 (talk) 00:21, 11 February 2013 (UTC)[reply]

Electrons (and positrons) have no physical size, so are always empty space. (However they do have electrical fields which extend for a distance.) There is no specific cutoff distance for when they annihilate - the probability just gets larger and larger the closer they are. They can even orbit each over for a while, see: Positronium. Ariel. (talk) 00:59, 11 February 2013 (UTC)[reply]
Joke: how many electrons and positrons can you fit in a lightbulb? Infinity. But seriously, Could you give a little more in your explanation? Thanks.68.36.148.100 (talk) 01:32, 11 February 2013 (UTC)[reply]
What do you want to know? Ariel. (talk) 03:01, 11 February 2013 (UTC)[reply]

"their probability space overlaps as they get closer, and eventually the likelihood of annihilation becomes great enough to cause it to happen, and it does." I don't mean to sound rude , but this sound like some south Philly used car salesman triple talk to me. Ariel, could you please explain this to me in other words?GeeBIGS (talk) 06:15, 11 February 2013 (UTC)[reply]

An electron does not have a defined position, but rather there is a chance of finding it anywhere. Technically it could even be light years away from where you would normally think of as it's "location" - but at an extremely low probability. That's what he means by "probability space". So when the electron and positron approach each other there is a chance that they actually happen to both be in the same space at the same time, but also a chance that they are not "near" each other. The closer they get the greater the chance of finding them both in the same spot. Once that happens they will annihilate. The visible effect of this is time. Instead of annihilating instantly they take some time to do it, and you can define a "half life" of sorts of how long it takes them to both randomly happen to be in the same place at the same time. But remember that it's random - for the same distance some will annihilate extremely fast, some will take longer. But you can calculate how long it will take half of them to annihilate. And of course the closer they are the shorter that time is - but it's never 0 and it's never infinity. I hope this helps. Ariel. (talk) 09:10, 11 February 2013 (UTC)[reply]

Define "in the same spot" and "the same place at the same time" and if you could please explain how both simultaneously ceasing to exist is not the same as being in the same space . GeeBIGS (talk) 12:00, 11 February 2013 (UTC)[reply]

"the same place at the same time" means their waveforms overlap such that the waves exactly cancel out. You can (if you want) define "the same space" as the location of maximum probability. But it's not necessary for those two locations (as defined that way) to be the same for them to annihilate. Ariel. (talk) 19:01, 11 February 2013 (UTC)[reply]
No, they don't cancel, since that would violate conservation of energy. There's no special rule about annihilation in quantum field theory. Decays of the form particle + its antiparticle → other particle + its antiparticle are allowed (if energy-momentum is conserved) simply because antiparticles have opposite conserved charges, so all the charges add up to 0 on both sides and hence are conserved in the decay. In particular, the photon is its own antiparticle, so particle + antiparticle → 2 photons is always possible (though it may not happen very often if the particle interacts with light only indirectly). -- BenRG (talk) 19:41, 11 February 2013 (UTC)[reply]

Mass of a test tube full of bare nuclei

I read this in a book A test tube full of bare nuclei will weigh heavier than the earth. If the test tube is heavier than earth, it means test tube contains more nuclei than the earth. So, tube is heavier than the earth. This also means that the density of test tube is more than the density of neutron star. This doesn't sound good to me, I think the fact (italic sentence) is wrong. What do you think about this? Show your knowledge (talk) 03:48, 10 February 2013 (UTC)[reply]

Are you sure you have that quote right ? It's bad English. Proper English would be "A test tube full of bare nuclei will weigh more than the Earth" or "A test tube full of bare nuclei will be heavier than the Earth". StuRat (talk) 03:54, 10 February 2013 (UTC)[reply]
I have written the same quote written in the book. Scientifically, I also think this quote is not right. What is your opinion about this? Show your knowledge (talk) 04:12, 10 February 2013 (UTC)[reply]
A test tube full of bare nuclei would be functionally as dense as a Neutron star, which has a density (at the lower estimate) of 3.7×1017 kg/m3 which means that a test tube full (say 5 cubic centimeters) would have a mass of 3.7×1017 kg/m3 x 5×10-6 m3 = 1.85 ×1012 kg. The earth has a mass of 5.9736×1024 kg, so no, it would not weigh more than the earth. It's 12 orders of magnitude lighter than the earth. But it's still really freaking heavy; a Boeing 787 has a mass of about 2.27×105 kg; so our test tube full of nuclei would still be heavier than 9,000,000 jumbo jets. --Jayron32 04:34, 10 February 2013 (UTC)[reply]
A typical test tube is actually more like 10cc's. But our Orders_of_magnitude_(mass) article (which is great for this kind of question) says that a teaspoonful (5ml) of neutron star material would have a mass of 5.5x1012...so I guess we might say 1013kg...but still nowhere near the mass of the earth. The orders of magnitude article suggest that a 1km tall mountain would have about the same mass as our test tube. About a thousand olympic-sized swimming pools full of nucleii would balance the mass of the Earth pretty well. SteveBaker (talk) 04:49, 10 February 2013 (UTC)[reply]
5.5... 1.85.... close enough for government work... ;) --Jayron32 05:06, 10 February 2013 (UTC)[reply]
So that quote is both wrong and bad English. I think I'd avoid that author. StuRat (talk) 04:50, 10 February 2013 (UTC)[reply]
I wonder if a pyrex test tube would be strong enough to contain a mass equivalent to millions of jet airplanes. ←Baseball Bugs What's up, Doc? carrots05:32, 10 February 2013 (UTC)[reply]
I wondered the same thing. I'm pretty sure the answer is no. That means we're talking about a strictly theoretical (= non-existent) test tube; and I don't see how we can have real nuclei sitting inside a non-existent test tube. So that must mean we're talking about theoretical (= non-existent) nuclei. In theory there's no difference between theory and practice; but in practice there kinda is, and this is a really good demonstration of it. -- Jack of Oz [Talk] 05:45, 10 February 2013 (UTC)[reply]
Well, it's not just the mass. What's keeping them squoze together? If they're "bare nuclei" other than neutrons, then the whole thing is positively charged and is furiously repelling itself while attracting surrounding electrons; I don't think you'd survive being anywhere near. If they're neutrons, then they have a half-life of about 15 minutes, meaning roughly 10000 times as radioactive as the Polonium-210 a microgram or so of which killed Litvinenko (in terms of disintegrations per unit time; admittedly each disintegration might be lower energy, but when you've got 10^26 times more of them I doubt that's going to help much). --Trovatore (talk) 08:21, 10 February 2013 (UTC)[reply]
Maybe only if it's really high quality pyrex. Although this strikes me as a half-cousin to the old question, "If you could develop a universal solvent, what would you store it in?" ←Baseball Bugs What's up, Doc? carrots06:21, 10 February 2013 (UTC)[reply]

This is great, I got the answer to my question in 1 hour. But I have already posted a question two days before (8 Feb, heading: Symbol of mass number) still I don't have the clear answer of that question. Please, do something about that. Show your knowledge (talk) 05:13, 10 February 2013 (UTC)[reply]

What's stopping you from googling it? ←Baseball Bugs What's up, Doc? carrots05:32, 10 February 2013 (UTC)[reply]
Sure, if you can get Wikipedia to double my pay for answering your questions, maybe I'll get on that. --Jayron32 05:34, 10 February 2013 (UTC)[reply]
Jayron32's pay rate is hereby doubled, so he will henceforth be paid twice as much per answer to Wikipedia Q as he was previously. StuRat (talk) 05:47, 10 February 2013 (UTC) [reply]
Strangely enough, my wallet doesn't seem any heavier. --Jayron32 06:01, 10 February 2013 (UTC)[reply]
In that case, I'd better triple your pay rate. StuRat (talk) 06:04, 10 February 2013 (UTC) [reply]
You'll get what's coming to you, I can guarantee you that, Jayron. -- Jack of Oz [Talk] 10:48, 10 February 2013 (UTC) [reply]
That's always what I am afraid of... --Jayron32 18:26, 10 February 2013 (UTC)[reply]
Isn't the issue here what exact meaning the word "full" has in this context? It talks about multiple nuclei, so that implies a least some spacing between them (if they were touching then wouldn't it just be one big nucleus?), and that spacing will by definiton determine the density. We normally say a test tube is full of water if any additional water would over flow, but we don't require the water to be compressed to qualify as being truly full. Conversely, a hydrogen cylinder is empty if it's pressure is equal to 1atm. 202.155.85.18 (talk) 00:31, 11 February 2013 (UTC)[reply]
I believe we're talking about the maximum density of neutrons it's possible to achieve outside of a black hole. Similarly to the core of a neutron star where the neutrons can't get any closer together due to quantum degeneracy pressure (ie the Pauli exclusion principle). True, our OP might be thinking of a less dense packing of neutrons - but what we're saying is that even at the densest packing imaginable, the test tube still only weighs as much as Snowdon - a modest-sized mountain. SteveBaker (talk) 16:04, 11 February 2013 (UTC)[reply]
I just realized...if you turned something with the mass of the Earth into a black hole, it would have a Schwartzchild Radius of about 1cm. That's a 2cm diameter - so at that density, it would just about fit into a large test tube...but not into a smaller one. SteveBaker (talk) 16:09, 11 February 2013 (UTC)[reply]

solar ac dc inverters failure

Is there a way to test an inverter to see if it is functioning properly, or do they just quit completely? My batteries are not lasting like they should and was told it could be the inverter. off grid207.212.113.253 (talk) 04:03, 10 February 2013 (UTC)[reply]

As a general, but not absolute, rule, electronics tend to fail catastrophically, or fail with some part getting far too hot, emitting a "brown smell". So, if your invertor has not failed completely, is not showing any warning/fault display, and is not emitting burnt smells, it's most likely not your problem. You can test an invertor by testing first with no load, then with something (such as an electric room heater) that loads it to at or near its full rating. Measure the DC input current for both conditions, and comnpare with the manufactuer's data. Typically the DC input amps should be roughly 1 twentieth the rated load in watts divided by the battery voltage. The full load DC input amps should be about 1.1 times the actual load divided by the battery voltage. Do not be concerned if the no load DC amps varies somewhat from this.
Are you aware that batteries are subject to both catastrophic failure and gradual loss in capacity? Battery life is dependent on brand quality, ambient temperature, the number of cycles, and the depth of discharge. In the hot climate regions of Australia, battery life for cheap European lead acid batteries can be as short as two summers. A life of ten years for a generously sized quality Janpanese battery is a good achievement. The output of the cheaper solar panels also deteriorates over time. A panel service life to 50% output of 10 years is not untypical. Sometimes the sealing fails and water gets into the silicon, causing an early drop in output. Keit 121.215.141.120 (talk) 05:50, 10 February 2013 (UTC)[reply]
In addition to Keit's excellent answer above, if you need to replace your batteries, you should be aware that the batteries sold for vehicle use are usually not suitable for long-term use with an inverter because they are not designed for deep discharge. Marine type batteries, or ones designed for your specific application should last much longer, but will cost at least twice as much (in the UK, at least). Dbfirs 08:40, 10 February 2013 (UTC)[reply]
IIRC, the "never add water" batteries which have become ubiquitous for automobiles these days (contain calcium, I believe) are much more susceptible to damage by deep discharge than the old "check and fill periodically" variety; my personal experience is that one instance of leaving the headlights on overnight or similar can make them essentially "dead", i.e. they may work under ideal circumstances but leave them for a couple of weeks, or the first cold day, and no juice. So like the guy says, don't use car batteries. Gzuckier (talk) 05:22, 11 February 2013 (UTC)[reply]

Do photons produce gravitational field ?

I know photons produce electric as well as magnetic field. I want to know, do they produce gravitational field ? I know electric field is always accompanied by magnetic field. Is it correct to say that "magnetic field is also always accompanied by electric field" ? There is another confusion, suppose if we increase the frequency of a photon, its energy also increases. If we increase the wavelength of a photon, its energy decreases. Since, E = hv. Give your response about last three sentences. Thanks! Parimal Kumar Singh (talk) 04:07, 10 February 2013 (UTC)[reply]

Photons certainly interact with gravitational fields, gravitational lensing, for example. Also, not every electric field necessarily produces a magnetic field. A magnetic field is generated by a moving electric charge. A stationary electric charge produces an electric field, but no magnetic field. I'm not sure I follow your confusion on your last three sentences. Frequency and wavelength are inversely proportional: if you get more vibrations, the space between them gets shorter; if you get less vibrations, the space between them is longer. So decreasing energy = decreasing frequency = increasing wavelength and vice-versa. --Jayron32 05:05, 10 February 2013 (UTC)[reply]
If an electric field is static, i.e., not moving or changing in strength, there is no resultant magnetic field as Jayron said. If the electric field is not static, there still will not be a magnetic field unless the field encloses at least part of an electric circuit - the magnetic field arises from the current in the circuit. Conversely, a magnetic field will not create an electric field if the space is a perfect electric conductor. In electromagnetic radiation, electric and magnetic fields propagate together. Wickwack 124.182.176.205 (talk) 05:36, 10 February 2013 (UTC)[reply]
Photons contribute to the energy density of a volume, which could be included in a generalized energy tensor used to calculate behavior using general relativity. In most scenarios, the effect is negligible because the energy-density of photons has a much smaller magnitude than the mass density of ordinary matter. Nimur (talk) 06:07, 10 February 2013 (UTC)[reply]

When I was reading about Albert Einstein, I found that photons have attraction towards gravitational field. Earth has mass and it produces gravitational field. Do photons produce gravitational field like our earth? Parimal Kumar Singh (talk) 10:54, 10 February 2013 (UTC)[reply]

Photons are massless, so they don't produce gravitational fields.Dja1979 (talk) 12:00, 10 February 2013 (UTC)[reply]
See Stress–energy tensor. Radiation can indeed be a source of gravitational fields in General Relativity. Jheald (talk) 12:20, 10 February 2013 (UTC)[reply]
They do produce a gravitational field due to the energy they have which depends on their frequency. If you had a box with mirrors it would weigh more if it had some light being reflected around inside it. Dmcq (talk) 12:25, 10 February 2013 (UTC)[reply]
To elaborate a bit on the above answers, note that one can always choose a reference frame such that an electron's magnetic field "disappears". It still exists however in other reference frames. Furthermore, the electric and magnetic fields are mediated by the energy of photons, the precise value of which is frame dependent in accordance with E = hv and the relativistic Doppler effect. Also important is the fact that mass and energy are equivalent and since gravity is a consequence of mass-energy, light produces gravity and it is also affected by it. -Modocc (talk) 14:41, 10 February 2013 (UTC)[reply]

This is a hugely perennial question at the Refdesk and just from the first 20 search results for "photon" and "gravity" I find [6] [7] [8] (also [9] which is useless). Of these, [10] appears to be the useful answer if you can access it. At some point it would be great if someone could write an actual article about photon gravity explaining this and any other good sources so we can send people there. There's some unexpected factor of 2 involved. Wnt (talk) 16:03, 11 February 2013 (UTC)[reply]

It's probably related to the factor-of-2 difference in the deflection of light by the sun, which is one of the most famous tests of general relativity. If so, it's more or less because spatial and temporal curvature contribute equally to the deflection when light is involved, while with nonrelativistic particles (and in the Newtonian limit) only the temporal curvature matters. I think. -- BenRG (talk) 19:25, 11 February 2013 (UTC)[reply]

Dumbed down Kindergarten level answer: Given that light is deflected by massive bodies, and given that photons have momentum, it follows from conservation of momentum that photons produce a gravitational field. Count Iblis (talk) 17:47, 12 February 2013 (UTC)[reply]

Dumbed down high school level answer, at the least. The average kindergartener is gonna give you a "huh?" when you're talking about "photons", "conservation of momentum", and "gravitational fields". So will many, perhaps most, high schoolers. —SeekingAnswers (reply) 12:48, 14 February 2013 (UTC)[reply]

Planets and frequency of sound

Dear Sir, this is not my homework

1) Which planet of solar system-
a) rotate faster
b) rotate slower
c) revolve around sun faster
d) revolve around sun slowerthan any other planet?
2) If i hear a sound of frequency 19,000 Hz, will it cause pain to my ears? What is relation between frequency and decibel?

Thank you. Walker — Preceding unsigned comment added by C. Walker19 (talkcontribs) 07:22, 10 February 2013 (UTC)[reply]

You will find the answers to 1) in the article Planet#Planetary_attributes. Please let us know if you need help interpreting this. Frequency is the pitch of a note, and 19 kHz is at the top of the hearing range, in fact most of the population cannot hear a note this high. If you are young, then maybe you can, especially if it is loud. Decibels are a measure of sound intensity or loudness. See the article Decibel for technical details. Any very loud sound can be perceived as pain and may cause hearing damage. Dbfirs 08:27, 10 February 2013 (UTC)[reply]
Agreed, and let me add one more tidbit. It takes less energy to produce high frequency sounds of a given volume than low frequency sounds of the same volume. Therefore, say 20 watts at 19 kHz may very well be painful, while 20 watts at at a lower frequency is not. StuRat (talk) 03:57, 11 February 2013 (UTC)[reply]

Tensor

I was searching for a simple definition of tensor, I tried to read the linked article, but it was beyond my knowledge. Then, a professor told me a simple definition - A physical quantity is said to be tensor if it is neither a scalar nor a vector as its direction is not properly specified, but has different values in different directions. Examples of tensor include strain, moment of inertia, density, refractive index, etc. Wikipedia article on tensor says just opposite - Vectors and scalars themselves are also tensors. Who is correct? Which one should I prefer? Sunny Singh 07:57, 10 February 2013 (UTC) — Preceding unsigned comment added by Sunnysinghthebaba (talkcontribs)

Oh, you've stepped in it now :-)
The simple answer to your question is that scalars and vectors are tensors, of rank 0 and 1 respectively, but you don't normally call them that, because why bother? Your professor was explaining tensors of rank 2 and beyond. --Trovatore (talk) 08:00, 10 February 2013 (UTC)[reply]
You might enjoy Dan Fleisch's "What's a Tensor?" video in which he uses "children's blocks, small arrows, a couple of pieces of cardboard and a pointed stick".[11] Sean.hoyland - talk 13:54, 10 February 2013 (UTC)[reply]
I'm biased due to my computer science background, but I think the simplest way to think about a tensor is as a multidimensional array. Imagine a one-dimensional array, say arr[32]. In mathematics, this represents a vector, which is a rank-1 tensor. Imagine a two-dimensional array, arr[32][32]. This is a rank 2 tensor, and every value arr[i][j] presumably has some physical meaning specific to the indices i and j. Similarly, arr[32][32][32] would be a 3-dimensional array and hence a rank-3 tensor, etc.
Here's a concrete example to help with your intuition. The stress-energy tensor describes the movement of four-momentum through spacetime. It's a rank 2 tensor, so you can imagine it as the array T[4][4]; it's 4x4 because spacetime has 4 dimensions. T[i][j], the jth column of the ith row of the array, is the flux of the ith component of momentum through the jth dimension. For example, since by convention the 1st dimension is x, T[1][1] is the flux of the momentum in the x direction, across the surface defined by constant x. Similarly, T[1][2] is the flux of the momentum in the x direction, across the surface defined by constant y. --140.180.243.51 (talk) 19:38, 10 February 2013 (UTC)[reply]
The problem with that formulation is that it's coordinate-specific (or basis-specific, depending on whether you're thinking of physics-style or algebra-style tensors).
To me a rank-two tensor is a machine that takes one vector and gives you back another. For the stress tensor, you can think about the input vector being the one that describes a little surface of a place you could potentially put a cut inside the object, and the output vector is how much pressure or shear the object puts on that surface. None of this needs any coordinate system to be specified, and you don't need to break it down in to components to understand it. (When you want to make a practical calculation, you probably will specify a coordinate system and take components, but you'll use whichever one is most convenient at the time rather than one specified in advance.) --Trovatore (talk) 23:50, 10 February 2013 (UTC)[reply]

Hydrogen Synthesization

Can hydrogen be synthesized by manipulation of subatomic particles? — Preceding unsigned comment added by Lawrie1 (talkcontribs) 13:56, 10 February 2013 (UTC)[reply]

Sure. Take one proton and one electron, bring them into contact: presto, you have a hydrogen atom. Looie496 (talk) 16:17, 10 February 2013 (UTC)[reply]
All of the science of chemistry could be described as "manipulation of subatomic particles" - but as Looie says, we can also do it directly from a stream of protons and a stream of electrons. SteveBaker (talk) 15:53, 11 February 2013 (UTC)[reply]

Power lines: buried or overground?

What are the advantages of one system or the other? Is it just the cost of implementing vs. the cost of maintaining? OsmanRF34 (talk) 16:06, 10 February 2013 (UTC)[reply]

It depends on how you define "advantages". Ruslik_Zero 16:47, 10 February 2013 (UTC)[reply]
"Underground cables take up less right-of-way than overhead lines, have lower visibility, and are less affected by bad weather. However, costs of insulated cable and excavation are much higher than overhead construction. Faults in buried transmission lines take longer to locate and repair. Underground lines are strictly limited by their thermal capacity, which permits less overload or re-rating than overhead lines. Long underground cables have significant capacitance, which may reduce their ability to provide useful power to loads." See our article Electric power transmission, also Overhead power line and Undergrounding. Alansplodge (talk) 16:52, 10 February 2013 (UTC)[reply]
There are also safety considerations that generally favor underground lines. Looie496 (talk) 16:57, 10 February 2013 (UTC)[reply]
Also note that there are two variants of underground power lines, those with access tunnels and those without. Most seem to lack access tunnels, and even go under streets, etc., requiring digging up those streets to access them. Those with access tunnels, while more expensive initially, make for far easier, quicker, and cheaper maintenance, upgrades, etc.
And, in the comparison of above ground to underground, we can't neglect that most people find above ground wires to be ugly. As such, they may bring down property values. StuRat (talk) 04:03, 11 February 2013 (UTC)[reply]
True, but the higher construction and maintenance costs of underground cables still appear to trump everything else. 24.23.196.85 (talk) 04:11, 11 February 2013 (UTC)[reply]
That depends very much on local conditions. In Germany, overland lines are above ground (as mentioned before, if you distribute AC, underground has significantly larger losses over long distance), but local distribution is nearly exclusively underground. It must be 30 years or so since I last saw an overhead line go into a private building. The cost advantage shifts based on population density, and also on Quality of Service. --Stephan Schulz (talk) 12:48, 11 February 2013 (UTC)[reply]

Breakdown of a Barrel of Oil

Oil is used to produce fuels and petrochemicals. I believe that more of the oil is used for petrochemicals and for fuels, and that an oil company's profits are derived more from petrochemicals than from fuels. Is there an entry that provides the (approximate) breakdown? 68.54.32.39 (talk) 17:39, 10 February 2013 (UTC)[reply]

A breakdown of the products made from a typical barrel of US oil.
Crude oil is what's taken from the ground and comes in barrels; it's described as sweet if it has low sulfur content which makes it easier to work with. The article gives average contents. It is then refined by heating and distillation and various other chemical processes that separate out and create different petrochemicals. So it's not so much that they just separate out what's already there as it is what they make from it. μηδείς (talk) 18:03, 10 February 2013 (UTC)[reply]
Most refineries use distillation to separate the crude oil into different length hydrocarbons. They also use fluid catalytic cracking to split long-chain molecules into more-in-demand shorter ones. It is also possible to merge short-chained hydrocarbons into longer ones, which might be useful if there is more methane that there is a demand for. CS Miller (talk) 19:44, 10 February 2013 (UTC)[reply]
I've added the article caption to the chart to make clear that the product listed are not the unrefined makeup of a barrel. I'm also moving this up to my post to save space--if you object, go ahead and revert the edit. μηδείς (talk) 03:54, 11 February 2013 (UTC)[reply]
Thanks for that by the way. Good call. --Jayron32 04:05, 11 February 2013 (UTC)[reply]
What are those percentages though? Percent by volume? By weight? By energy content? Either way, that wouldn't answer the OP's question which is "By dollar value". The original WP:RS upon which that chart is based [12] is similarly unclear. Any chart with unlabelled axes is essentially useless!! SteveBaker (talk) 15:44, 11 February 2013 (UTC)[reply]
Not to mention, that EIA chart indicates refinery output, not refinery output per barrel of oil. A significant amount of refined product is re-run through the refinery; so for example, if your refinery starts with one barrel of crude, you might produce twenty gallons valuable gasoline and twenty gallons of low-value heavy asphaltenes; so you sell what you can and dump the "waste" asphalt back in with the next barrel of crude. From that, you might produce twenty-two gallons of gasoline and ... twenty more gallons of asphalt and miscellaneous products. (Hopefully not in those ratios!) The real world of refinery chemistry is not as straightforward as a pie-chart makes it look! Here's my favorite refinery poster (the one I used to hang on my wall for reference): Mustang Engineering's Modern Refinery, one of many excellent promotional posters they produce. (Though, the PDF is no longer available online, you can order a copy from them). Nimur (talk) 05:47, 12 February 2013 (UTC)[reply]

<-I'm not sure the premise of this question is entirely valid. It's probably fair to say that to a first approximation, oil company income comes from finding and producing hydrocarbons (i.e. their "upstream" actvities) rather than doing things to those hydrocarbons (i.e. their "downstream" activities). Obviously it's a lot more complicated than that in reality and each company is different in terms of the upstream vs downstream activities/profitability, but Exxon Mobil earnings for 2011 after tax for example were ~41 billion dollars, ~34 of which came from their upstream operations. Sean.hoyland - talk 04:27, 11 February 2013 (UTC) As original poster of the question, I thank y'all for the inputs. Right, a simple question but the answer is complex. 68.54.32.39 (talk) 20:41, 12 February 2013 (UTC)[reply]

Upright exercise bike question

What should be the position of the knees relative to the feet while riding an upright stationary bike? I mean should the knees be in the same level of the feet (distance between feet = distance between knees), or the knees be slightly bent outwards (distance between feet < distance between knees)? Which is the correct posture to prevent knee injury? Is 18 inches distance between the outer edges of the the two feet when they the over the pedals a safe distance? --PlanetEditor (talk) 17:47, 10 February 2013 (UTC)[reply]

The knees should be in the same plane as the feet, as nearly as possible. I can't speak to specific distances, but generally you want your knee bending in the way it naturally bends, not in any other direction. Riding significantly bowlegged is bad. Looie496 (talk) 19:44, 10 February 2013 (UTC)[reply]
Thanks. --PlanetEditor (talk) 03:01, 11 February 2013 (UTC)[reply]

Asteroid 2012 DA14 and orbital resonance

After asteroid 2012 DA14 passes by the Earth in a few days, its orbital period will be 317 days. That is very close to a 7:6 ratio with Earth's orbital period. Is this significant - i.e. an orbital resonance? Bubba73 You talkin' to me? 19:39, 10 February 2013 (UTC)[reply]

A resonance is a result of many small perturbations accumulating to give a stable relationship. In this case the 317 day period will result from a single huge perturbation, the previous period being 368 days, so it couldn't possibly be a resonance. Generally speaking Earth's gravity is too weak in comparison to Jupiter's for Earth to be able to produce stable orbital resonances. Looie496 (talk) 19:51, 10 February 2013 (UTC)[reply]
Thank you
Resolved
Bubba73 You talkin' to me? 20:27, 10 February 2013 (UTC)[reply]
The exception being resonances with very low denominators - particularly 1:1 resonances. See horseshoe orbit for instance. --Tango (talk) 19:00, 11 February 2013 (UTC)[reply]
99942 Apophis is affected by the Earth, isn't it? Bubba73 You talkin' to me? 00:39, 12 February 2013 (UTC)[reply]
Certainly, but not to the extent of being in resonance. Everything in the (observable) universe is affected by everything else to some extent. --Tango (talk) 12:27, 12 February 2013 (UTC)[reply]
Whoops, I actually meant 3753 Cruithne. Bubba73 You talkin' to me? 01:52, 13 February 2013 (UTC)[reply]

Dr. Imenson (spelling?) medical doctor

Dr. Imeson (not sure of the spelling) was my family doctor in the late 1940's and into the 1950's. I would like any information on him. What happened to him. When he passed on, etc.. — Preceding unsigned comment added by 24.7.138.9 (talkcontribs) 17:25, February 10, 2013‎

I need to add that he was a doctor in San Francisco, California — Preceding unsigned comment added by 24.7.138.9 (talkcontribs) 17:28, February 10, 2013‎

There are plenty of people-search websites you could try. A normal Google search turns up the following possibility:
Dr. Shale Imeson, MD Anesthesiologist, Pain Management Physician in Stockton, CA. However, since he's still around, it is unlikely that he was a Dr. in the '40s-'50s -- Maybe his father?    ~:74.60.29.141 (talk) 23:46, 10 February 2013 (UTC)[reply]

February 11

Anodisation of Aluminium

Ive read numerous stories of iphone 5s being scuffed from the moment they opened the box. Ive also heard similar stories with other anodized aluminium products. This leads me to wonder, is anodisation a process which is difficult to achieve without causing small scuffs or marks? Clover345 (talk) 00:13, 11 February 2013 (UTC)[reply]

Not at all, but aluminum is a relatively soft metal, compared to say, stainless steel which I think was used on some of the earlier iPhone models. Metals of high hardness scratch less easily (and a scuff is basically just a scratch as I understand the term). 202.155.85.18 (talk) 00:17, 11 February 2013 (UTC)[reply]
One of the advantages of traditional anodising done with electric current per standards is that it provides a surface significantly harder than the base aluminium. However, it is a porus surface, further treatment (eg varnish) is usually done if the device is to be handled. See http://en.wikipedia.org/wiki/Anodizing. There could be a production problem with the further treatment. Also, there are various non-electric processes that result in what looks like an undyed anodised surface - these are not as tough. A substitute finish used by a company I once worked for used the following (much cheaper) process: Step 1 - clean with detergent and air blower dry. Step 2 - dip in dilute caustic soda. Step 3 - clean again with citrus solvent (slightly acidic). Step 4 - water rinse and air blower dry. Step 5 light spray with thinned marine varnish. The result looks identical with genuine anodising, but has no where near the toughness and durability - and usually the product left the shop with marks already on it. Ratbone 121.215.57.135 (talk) 01:06, 11 February 2013 (UTC)[reply]


Aluminum is relatively soft, but an anodized surface on aluminum is very, very hard. this page claims that apple uses something called "soft anodizing" which I am unfamilar with. --Guy Macon (talk)

Betretta M9 hammer hole

What is the purpose of the hole in the hammer spur of the M9 Baretta?

202.155.85.18 (talk) 02:42, 11 February 2013 (UTC)[reply]

Having less weight on the hammer means it can move faster, reducing the time between pulling the trigger having the gun fire. I think it is also done for aesthetic reasons. --T H F S W (T · C · E) 04:54, 11 February 2013 (UTC)[reply]
Maybe it's a design feature to match the seemingly useless hole at the bottom of the handle. ←Baseball Bugs What's up, Doc? carrots12:09, 11 February 2013 (UTC)[reply]
The "useless hole" at the bottom is for a lanyard Bugs. Shadowjams (talk) 05:35, 12 February 2013 (UTC)[reply]
Not necessarily useless - ISTR some soldiers (or rather, officers) used to have a piece of string attaching their handgun to their belt (or holster, or similar). The hole at the bottom of the handle is in the perfect place for doing this. --Demiurge1000 (talk) 12:48, 11 February 2013 (UTC)[reply]
See Lanyard#Origins. No idea on the hammer spur, though. --Dweller (talk) 14:55, 11 February 2013 (UTC)[reply]
Nobody in their right mind puts a lanyard on a hammer. Shadowjams (talk) 05:34, 12 February 2013 (UTC)[reply]
I was replying to a comment that read "The hole at the bottom of the handle is in the perfect place for doing this". If there had been any doubt that was what I meant, the second of my two brief sentences should have addressed it. --Dweller (talk) 09:06, 12 February 2013 (UTC)[reply]
And you apparently take indents a bit too personal. Shadowjams (talk) 10:31, 12 February 2013 (UTC)[reply]
As Demiurge was also clearly talking about the hole at the bottom of the handle, your impersonal comment seems to have been aimed at no-one at all. --Dweller (talk) 11:11, 12 February 2013 (UTC)[reply]
It's called a "skeletonized hammer". This means the gun has the same hammer geometry as it would with a solid hammer, but the hammer is lighter. Beretta themselves say here that this "can improve cycle time because of the lighter weight." Alternatively, as the hammer is lighter, you can replace the hammer spring with a weaker one (a lighter hammer is less mass for the spring to accelerate). This gives the gun a lower "pull weight" (the amount of force needed to pull the trigger); a smaller shooter (with weaker fingers) might opt for a lower pull weight, so they can still operate the gun easily. Naturally a lower pull weight isn't without issue, as a more sensitive trigger can make it more likely that the gun is fired accidentally. One can also get skeletonized triggers, which again lower the mass of the moving trigger system. -- Finlay McWalterTalk 16:23, 11 February 2013 (UTC)[reply]
Interestingly, I've never come across this question before, but it's a good one. The common theme is that most match hammers are milled like this... compare most standard 1911s to a match-grade 1911, and you'll see similar milled out hammers. My guess is it has to do with weight. It doesn't have to do with trigger pull... that's easily adjustable through other means, and the weight of the hammer matters almost nothing for that. Yes that gun's a DA, but you see this on single action guns all the time. The factory 92FS hammer is not referred to as "skeletonized" either, as the link clearly indicates. The hammer in the link is very different than the textured hammer in the picture.
I don't have a satisfactory answer for the OP, wish I did, because it's a good question. I can dispel, however, a few obviously wrong answers. Shadowjams (talk) 05:32, 12 February 2013 (UTC)[reply]

hair drier off lithium batteries?

can a hairdryer (very high draw device) be run off of any common, relatively light-weight batteries? The usage case would be that it is OK to charge for a long time, but then a very high drain hairdrier should deplete it in just a minute or two... are any battery technologies appropriate? If the battery would get warm, the hairdrier can actually draw air from over it (preheating the air) so maybe this helps a bit. The application is actually on an RC helicopter that is supposed to work in a very new way. The battery for the hairdrier can be separate. Note: this is just about a special application, like an rc helicopter that can help glue sheet paneling. Thanks! 91.120.48.242 (talk) 12:32, 11 February 2013 (UTC)[reply]

Google [battery operated hair dryer] and a number of entries turn up. ←Baseball Bugs What's up, Doc? carrots12:42, 11 February 2013 (UTC)[reply]
Uh, this is a pretty resounding "no". Anyway, nobody uses a hairdrier for "just a few seconds" so I'm wondering if you can answer my quesiton more directly. (The hairdrier was a stand-in anyway.) Can a battery release 1600 watts for, say 30 seconds at a time, without damage or terrible overheating? I mean, long-charge, fast-discharge batteries... does that describe the profile of existing, compact, light-weight batteries? thanks... 91.120.48.242 (talk) 13:33, 11 February 2013 (UTC)[reply]
Probably not; a hair drier pulls an intense amount of current, and there's just not enough electrons at a high enough energy in a small, compact lithium ion battery to power such a devise for a meaningful amount of time. Battery-operated hair driers could possibly run off of the sorts of rechargable batteries used for cordless drills and tools like that, but only if you were just running a powerful fan motor. The heating element is the real energy hog in the hair drier, and you really need a huge amount of electrical power to run such a heating element effectively. The chemistry just doesn't work out given the amount of material available in any compact chemical battery. --Jayron32 16:41, 11 February 2013 (UTC)[reply]
Try googling the subject further and see what you find. ←Baseball Bugs What's up, Doc? carrots13:42, 11 February 2013 (UTC)[reply]
RC hobbyists sometimes want fairly high discharge rates and they usually rely on lithium polymer batteries for the purpose although what you're asking for seems a bit extreme to me. It may be barely doable but it depends greatly on what you mean by 'relatively lightweight'. For example our article mentions some batteries managing up to 65C [13] continuous. As an example it links to [14]. If the specs are correct, this battery can handle up to 292.5A continuous discharge. With the nominal voltage of 37V (it's 10S) this means it should be able to supply 10,822.5W although given the rate of 65C this will be for less then a minute. However the battery itself is over 1.3kg so may not be considered lightweight. (I have to admit I wonder if the cabling and battery is really capable of handling 292.5A but perhaps it can.) Of course you don't need quite that level of power. So if we look under the category and sort by weight, you can find [15] which is 2200 mAh which given the rate of 65C can handle up to 143A continuous discharge. With the nominal voltage of 11.1V (it's 3S), this will be 1,587.3W so may fit your purposes. The battery is 266g. Of course you don't really need such a high discharge rate, more cells in series with a lower discharge rate would be fine. For example you could also use [16] which at 6S and 40C is supposed to be able to handle up to 72A continuous discharge so should be able to supply 1,598.4W for slightly longer then a minute although it's heavier at 385g. Probably one problem is generally for RC purposes, if you want a high voltage it's generally for a bigger device so a heavier battery isn't such a big deal. Of course the other point is even at 10S, you're still going to need about 43A. Personally I wouldn't want a lithium polymer battery supplying 43A near me (and if it's supplying 43A to a hair dryer it's likely to be near me) or I may find my hair getting dry a lot fast then I intended, along with the rest of my body. (While RC batteries are subject to shocks a hairdryer hopefully won't be, there are plenty of videos showing what can go wrong.) Nil Einne (talk) 17:00, 11 February 2013 (UTC)[reply]
It sounds like you don't actually want a hairdryer for heating hair but even so I'd suggest that even if it's theoretically possible, it's not practical for your purposes. These sort of things are at the bleeding edge (particularly when pushed close to the limits of the specs) and are targeted at a hard core group of hobbyists willing to take the associated risks (and as I said, there are plenty of videos and forums posts to show it). And in fact given your purposes it sounds like you will be in the group where 'rough handling' does come in to play. And since it sounds like this has some sort of industrial purpose, I quite doubt OSH and the fire brigade or the equivalent in your jurisdiction will be happy about the occasional exploding battery while charging, landing or storage (after a crash for example). Nil Einne (talk) 17:22, 11 February 2013 (UTC)[reply]
A look in RC forums suggest most 'C' ratings are, shall we say, optimistic, particularly when coming from HK and Chinese sellers. However this rather old post [17] suggests 40C may be possible or even 46C (120A in the case in point) although that was only with a single cell of a 3S battery. Of course even if all 3 could supply that at the same time you'd still be about 400W short, you'd need 4S. I didn't find the weight but it was suggested to be typical given the capacity (3S, 2600mAh) so from what I saw earlier it'll probably be under 400g.
In any case, I'd note you only seem to be considering this 'hair dryer' load. If you're planning some sort of RC helicopter, you'd also need power to lift the helicopter itself along with the battery and !hair dryer, take it to the destination, stay there while you glue the boards and then land again. It sounds quite unlikely 30s will be enough for this, so you'd need to adjust your demands and requirements. Presuming you come up with something like 3-5 minutes total flying time and you don't need more then say 300 watts for the helicopter (including all loads), from what I've already seen from over 1.5 years ago, it does sound likely it may be possible. I still think it isn't practical for anything other then demonstration/we can do this purposes. Edit: And I also question what you'll actually achieve if you only heat the glue for 30 seconds and then need to charge your helicopter or send a new one. Notice also the comments on cycle life and heat.
Nil Einne (talk) 18:19, 11 February 2013 (UTC)[reply]
A discharge rate that fast might be closer to the capabilities of a capacitor than a battery, specifically a motor capacitor. Note that such devices are dangerous, as the voltages produced may match or exceed mains voltage. StuRat (talk) 16:33, 11 February 2013 (UTC)[reply]
Sturat, this is very good/interesting. I didn't know this before. Do you think it's possible to have one that is really very much "capped" (hehe) at 1600Watts, and will discharge for, say, a full 1 minute, if it has 1 minute * 1600 Watts in it for the voltage that's being drawn? Thanks. 91.120.48.242 (talk) 16:43, 11 February 2013 (UTC)[reply]
Stu is correct that a capacitor would provide a large current for a short time, but you are very unlikely to find a capacitor that you can afford with enough capacity to run a hair dryer heating element for more than a second or two. A very big capacitor might run a small DC fan motor for a minute or so, but would gradually reduce in speed as the capacitor discharged. I'd advise running a 12v hair dryer from a car booster battery, not lithium. Dbfirs 17:09, 11 February 2013 (UTC)[reply]
Nil's suggestion above, of a lithium polymer battery, sounds like it suits your purposes better than a capacitor, although it's still dangerous. StuRat (talk) 17:32, 11 February 2013 (UTC)[reply]
It would help if you could describe the purpose of this device more. Are you trying to dry paint on an high wall ? If so, perhaps shining lots of heat lamps at it from the ground might be a better approach. StuRat (talk) 17:38, 11 February 2013 (UTC)[reply]

I think you need to channel the antimatter emitter directly into the output stream... no, really, what I'd wonder is, is there a way that you can store the energy chemically in the glue itself, rather than electrically? But I like the direction of your mind. :) Wnt (talk) 17:40, 11 February 2013 (UTC)[reply]

From the OP's previous posts, I do wonder the state of the OPs mind when they had this idea. Nil Einne (talk) 18:21, 11 February 2013 (UTC)[reply]
To get your heat it could be easier to burn a liquid fuel like kerosene. Your storage can be much lighter weight. Graeme Bartlett (talk) 21:02, 12 February 2013 (UTC)[reply]
Exactly. We've run up against the reason we're not all driving electric cars, despite their numerous advantages, and why electric airplanes are restricted to model planes; which is that a tank of hydrocarbons holds so much more energy than the best battery of similar weight and/or size as to make it no contest. For this application, note that one of the interchangable tips on my little butane soldering pencil (from radio shack, so many years ago) is a catalytic tip which generates a nice flow of hot air, suitable for heatshrink tubing etc., without the dangers of a flame. Gzuckier (talk) 05:12, 13 February 2013 (UTC)[reply]

I strongly suspect that our OP - having been defeated in his efforts to come up with a way of having an R/C plane transform into a helium balloon when it's batteries run down (see previous questions) - is now trying to transform it into a hot air balloon. Still not going to work - still going to be blown so far off-course while it's batteries recharge that it'll never get back on course before they run down again...etc, etc. 21:47, 12 February 2013 (UTC)

Resonant frequency of an aluminium bar.

Hi! I'm trying to get a rough idea of the resonant frequency of an aluminium bar that's 1400 x 40 x 40mm and can probably be considered to be "unclamped" - in the plane that I'm expecting the vibration to occur.

I don't need an exact answer - I'm just trying to find out whether it's likely to have a resonant frequency somewhere between 10 and 20Hz.

SteveBaker (talk) 14:03, 11 February 2013 (UTC)[reply]

Hmmm. Looks like the speed of sound in aluminum is roughly 5100 m/s. So sound travelling 1.4 m and back should take 2.8 m / 5100 m/s = 1800 Hz. The other directions should be higher. Acoustic resonance says more about this. My perception is that such disturbing "infra"sound can come from ducts less than twice this diameter, but this is presumably because of some lengthwise geometry, that the speed of sound in air is 15 times slower and perhaps because for some reason the formula for a closed tube is /4L instead of 2L. Wnt (talk) 15:50, 11 February 2013 (UTC)[reply]
The bending vibrations of the bar will be at a much lower frequency, http://hyperphysics.phy-astr.gsu.edu/hbase/music/barres.html may be helpful (I haven't checked it thoroughly). Note that when you clamp a free free bar the first bending mode drops in frequency. The reason is subtle. Greglocock (talk) 01:57, 12 February 2013 (UTC)[reply]

Atom Volume

I can't seem to find any references for the assertion that "approx. 99.9% of an atom's volume is empty space". Can someone please provide some references, or is this assertion not verified? Thanks.165.212.189.187 (talk) 16:55, 11 February 2013 (UTC)[reply]

Which article says that? Dmcq (talk) 17:58, 11 February 2013 (UTC)[reply]

I don't think any do.165.212.189.187 (talk) 18:02, 11 February 2013 (UTC)[reply]

I googled 99.9% of an atom's volume is empty space and got a lot of hits, some of which at a glance seem to be legit. Duoduoduo (talk) 18:23, 11 February 2013 (UTC)[reply]

Anywhere in Wikipedia?165.212.189.187 (talk) 18:36, 11 February 2013 (UTC)[reply]

It depends on what is meant by "empty space". If you consider electrons to be point particles, like little tiny infinitesimal balls, and the nucleus to be another tiny, solid ball, then I suppose the claim could be made. But that's a rather primitive and not-very-accurate view of what an atom is. Instead, if you consider an atom to be a nucleus surrounded by the probability space which contains the electrons, then it's as solid as anything; if by solid you mean "impenetrable little hard nugget", then an atom is essentially solid, and none of it is "empty". The entire volume of any given atom is essentially all made up of the electron cloud, which is essentially impenetrable by other atoms. So the "claim" that atoms are "empty space" is based on some rather silly leaps of logic from a non-too-accurate view of what an atom is. It sounds all profound and all, but its basically bullshit. --Jayron32 18:41, 11 February 2013 (UTC)[reply]
You can take it the other way too. At atom can be considered 100% empty space. The electrons are point like particles with no size, the nucleons are in turn made up of quarks which are also point like. So in some way an atom is 100% empty space. Of course in the real world atoms interact not by their location, but by the forces acting on them and those forces have ranges. So the forces acting on the electrons certainly reach the nucleon, and the entire area in between can be considered to be "filled in" with that force. Same for the strong force holding the nucleons together and also for the force in between the quarks. The interesting conclusion is that the "size" of particle depends on what force you want to use to measure it with. Ariel. (talk) 18:51, 11 February 2013 (UTC)[reply]
That's why any visualization you can conceive of to represent the atom or its parts always falls woefully short of how it actually behaves. See the discussion a few days ago regarding this very topic. --Jayron32 19:29, 11 February 2013 (UTC)[reply]
One sense in which you can say they're mostly empty is mass/energy density. -- BenRG (talk) 22:11, 11 February 2013 (UTC)[reply]

Duo, anywhere in wikipedia??165.212.189.187 (talk) 20:08, 11 February 2013 (UTC)[reply]

So is it true or not??165.212.189.187 (talk) 15:20, 12 February 2013 (UTC)[reply]

The best answer we can give you is it isn't even wrong. The premise upon which it is built: that you can reliably model an atom as a collection of relatively solid parts, with the balance being empty space, isn't correct in any way. So, there's not a simple "yes or no" answer; though if you want to know about some of the common models of the atom, and want to know why it is so hard for us to give you a straight answer, read the responses in detail with the understanding that concepts like "empty space" and "particle" and the like don't make sense on this scale, and instead you need to very carefully define your terms before the question becomes meaningful. --Jayron32 18:48, 12 February 2013 (UTC)[reply]

Even if you focus soley on the location of the electron and disregard the momentum so that you "find" its location?165.212.189.187 (talk) 19:14, 12 February 2013 (UTC)[reply]

Logic

Is this a true statement?: Things that don't exist have the same quantum state.165.212.189.187 (talk) 17:00, 11 February 2013 (UTC)[reply]

If coded in Predicate logic and following its usual axioms then that would evaluate as true. Basically what we'd be asking is whether the statement for all x and y, do x does not exist and y does not exist imply property 'quantum state the same', is that falsifiable ie can we find x and y so it is provably false? However this is an empty meaningless statement and the answer I gave is one more suited to the maths reference desk. Dmcq (talk) 17:49, 11 February 2013 (UTC)[reply]
More concisely: Everything exists, so any sentence starting with things that don't exist have... is vacuously true. Falisfiability and provability are unnecessary distractions. --Trovatore (talk) 18:59, 11 February 2013 (UTC)[reply]
Yes, that's the heart of it. We can also have fun with the vacuous truth of conditionals that have false antecedants. For example this sentence: "If this question is posted in January, then Wikipedia is an invisible pink unicorn" -- is true. SemanticMantis (talk) 19:22, 11 February 2013 (UTC)[reply]

Yes, good one, I can't wait until next year. or, since EVERYTHING exists then so does an Invisible Pink Unicorn.68.36.148.100 (talk) 01:54, 12 February 2013 (UTC)[reply]

No, you're missing the point. Everything exists. That's a true statement, but a less interesting one than it appears — it's actually a sort of tautology, because every thing is a thing, and things, by virtue of being things, must exist.
On the other hand, "a square circle exists" is a false statement. But there is no contradiction, because to conclude from "everything exists" that a square circle exists, you would first have to have a square circle to instantiate the universal quantifier "everything". --Trovatore (talk) 03:23, 12 February 2013 (UTC)[reply]

Still can't wait til next year. So no thing doesn't exist, but does nothing exist also?68.36.148.100 (talk) 04:10, 12 February 2013 (UTC)[reply]

Nothing doesn't exist. There's nothing you can do that can't be done. There's nothing you can sing that can't be sung. --Trovatore (talk) 04:15, 12 February 2013 (UTC)[reply]

The electron and positron existED once then annihilated each other getting just close enough to not exist forever? Then we must qualify what things exist with a time coordinate in order to be true or false right?68.36.148.100 (talk) 04:31, 12 February 2013 (UTC)[reply]

All very interesting thanks for your honest input. Does Empty space exist?165.212.189.187 (talk) 19:36, 11 February 2013 (UTC)[reply]

Yes, at least conceptually. So you can make meaningful statements about it. --Stephan Schulz (talk) 19:45, 11 February 2013 (UTC)[reply]
Interesting, I thought the current most correct answer to the question "does empty space exist?" was NO. Vespine (talk) 21:46, 11 February 2013 (UTC)[reply]

Thanks vespine, even more of a reason to include vacuum space as a critical component of the atom. There might be more going on in there than doesn't meet the eye. No?68.36.148.100 (talk) 01:18, 12 February 2013 (UTC)[reply]

  • No, the statement is neither true nor false, but ambiguous. The correct statement is that "things that don't exist have no quantum state." μηδείς (talk) 03:09, 12 February 2013 (UTC)[reply]
    • Because it's the empty set; and while there are endless ways of grouping elements into different sets, there is only one set that contains no elements - namely, the empty set. ←Baseball Bugs What's up, Doc? carrots03:22, 12 February 2013 (UTC)[reply]
      • Actually, ambiguous is probably too weak a term. Could you specify the quantum state of an atom with an atomic number of 500, and claim it has the same quantum state as a hydrogen atom inside a flask which contains only oxygen molecules? They'd both be descriptions of non-existent things. Things that don't exist don't have any identity at all--you can't then claim they do have the same identity. My not being a fire breathing dragon does not in any way make me the same as subtracting 37 from 51 while underwater or the dress size of last night's bad dream which are also not fire breathing dragons. μηδείς (talk) 04:35, 12 February 2013 (UTC)[reply]

I don't think your last point is relevant.68.36.148.100 (talk) 05:34, 12 February 2013 (UTC)[reply]

But what about things that once did exist but now don't, that is categorically different than "never proved to have ever existed", or "figment of your imagination" because you can describe it as having existed prior to X time, which is part of its identity forever, is it not? Once you designate "existing-thing-to-non-existent-'thing'" the description is precisely and only that, and it is exactly the same for all things that exist then cease to exist.165.212.189.187 (talk) 15:15, 12 February 2013 (UTC)[reply]

The passage of time doesn't "erase" existence as if something never had existed. You just have to keep your tenses straight, and you'll notice that the language I just used is perfectly common English. But you cannot say that the current King of France is the same as a three-inch flying unicorn because they both don't exist. μηδείς (talk) 02:02, 13 February 2013 (UTC)[reply]

This: The passage of time doesn't "erase" existence as if something never had existed. is MY point. But to further my point notice the "used-to-exist" vs. "figment of imagination" argument. You said yourself that a detailed description of non-existent things don't define those "things". I would liken your use of current king of France and the 3 inch unicorn to gibberish and then say that they are both categorized as such. 165.212.189.187 (talk) 15:37, 13 February 2013 (UTC)[reply]

Geological change in recorded history

It is a known fact that Earth has undergone continuous geological changes throughout its history and the process is ongoing. I'm wondering whether there have been any visible geological change in recorded history? --PlanetEditor (talk) 17:04, 11 February 2013 (UTC)[reply]

I saw a documentary a while ago about post-glacial rebound. They interviewed an old woman, who had lived her whole life in the same place (it was either in northern Sweden or in Finland). They went to the house she had lived in as a child; she said that then the family rowing boat was tied up right outside the window. Now the ground had risen such that the water's edge was about 10 metres (horizontally) from the house. This paper gives the rate of rebound in Skellefteå at nearly 1cm/year; given that there was maybe 80 years between the woman's early recollection and the documentary being made, that's consistent with the land on which her house stood being maybe 80cm higher than when she was a little girl. This is a geological (and not just a local) phenomenon, as all of Britain and Fennoscandia is tilting in this manner (the woman just lived somewhere where the effect was particularly high, and particularly evident). So that's not just recorded history, that's living memory. -- Finlay McWalterTalk 17:30, 11 February 2013 (UTC)[reply]
Changes due to Earthquakes and Volcanoes qualify, don't they? Mingmingla (talk) 17:44, 11 February 2013 (UTC)[reply]
I don't think so. Volcanoes, for example, have only limited effect in a limited region without significant and large-scale stratigraphcal change. In that sense, mining activities by humans will also count as geological change. --PlanetEditor (talk) 18:01, 11 February 2013 (UTC)[reply]
Geological change happens at all scales from planetary to atomic. Sean.hoyland - talk 18:09, 11 February 2013 (UTC)[reply]
Will 1964 Alaska earthquake qualify? The earthquake was accompanied by vertical displacement over an area of about 520,000 square kilometers. .... Vertical displacements ranged from about 11.5 meters of uplift to 2.3 meters of subsidence relative to sea level. Off the southwest end of Montague Island, there was absolute vertical displacement of about 13 - 15 meters. .... This zone of subsidence covered about 285,000 square kilometers, including the north and west parts of Prince William Sound, the west part of the Chugach Mountains, most of Kenai Peninsula, and almost all the Kodiak Island group. [18] Ruslik_Zero 19:00, 11 February 2013 (UTC)[reply]
Even more dramatic, and more recent, was 2004 Indian Ocean earthquake and tsunami. ←Baseball Bugs What's up, Doc? carrots01:51, 12 February 2013 (UTC)[reply]
Thank you guys for clearing my misconception. --PlanetEditor (talk) 02:37, 12 February 2013 (UTC)[reply]
More volcanic changes: 1883 eruption of Krakatoa, check the map in the infobox. Surtsey didn't exist before 1963. Today it's the size of a small village. The 1980 eruption of Mount St. Helens took out the top and one face of the mountain. There's some nice before-and-after pictures in the article. The Minoan eruption of Santorini is a likely source of the Atlantis legend. Look at the map: That used to be one complete island, similar to Krakatoa. Look at the Hawaiian Islands. Those are merely a string of islands created by the Pacific plate dragging across a giant volcano which has been erupting more-or-less constantly for millions of years. The Big Island's shape has changed considerably over the past few millenia. --Jayron32 04:46, 12 February 2013 (UTC)[reply]
In fact, the crater at Halemaumau has changed its shape in the last thirty years. We have photographic evidence of it being a different shape prior to volcanic activity in the 1970s and 1980s. And we have drawings and records from the previous century that describe and depict even more dramatic shape-changes. Perhaps some of the most stunning video-recordings are the eruptions of the 1950s, culminating with the 1959 eruption in which thousand-foot-tall mountains of lava burst out of the ground. We also have an article on the former island Jolnir, and on the cornfield-volcano Parícutin; and even non-seismic geological events like the subsidence of California's central valley... and the list goes on. There's no shortage of geological change that has happened fast enough for humans to watch and photograph and record it! Nimur (talk) 06:05, 12 February 2013 (UTC)[reply]
  • The course of the Nile has changed over the centuries and its navigable mouths have had to be resurveyed and charted.
  • The Tigris and Euphrates used to flow separately into a much longer Gulf of Arabia which has silted up over the millennia.
  • The course of various rivers such as the Mississippi River have changed so much in recent centuries that bits of land from various states have ended up on the other side of the river from their mainland. See New Madrid Earthquake.
  • Iceland when the Vikings arrived and much of the Levant even during Roman times was green or forested until goats got to them.
  • According to the oral history of the Nivkh people, Sakhalin Island was a peninsula until well after the founding of the Chinese Empire, when rising sea-level broke through at the Amur River outlet, severing it from the Eurasian mainland.
  • The town of Calais used to be English til the French had it towed to the other side of the Channel by Napoleon.
μηδείς (talk) 02:56, 12 February 2013 (UTC)[reply]
Is the last one really true? Sagittarian Milky Way (talk) 04:31, 12 February 2013 (UTC)[reply]
Is your lack of a sense of humor real? --Jayron32 04:38, 12 February 2013 (UTC)[reply]
Of course it's nonsense! Everyone knows that Napoleon lived in the 18th and 19th century, while Calais was towed over to France under Henry II in 1558. France didn't even have real galleys for the towing around 1800. But you can still see where Calais broke off. --Stephan Schulz (talk) 11:05, 12 February 2013 (UTC)[reply]
Jist wannit a make sher youse guys was payin a tension. μηδείς (talk) 21:00, 12 February 2013 (UTC)[reply]
There was a hell of a lot going on in Chile in 1960 (don't be fooled by the title, it was way more than just one event). I still remember news reports, saying the entire Chilean coast would have to be redrawn. -- Jack of Oz [Talk] 02:10, 13 February 2013 (UTC)[reply]

Splitting water with a vacuum

Suppose you have a small icy asteroid you'd like to terraform. You wrap it securely in a watertight membrane that reflects infrared light, you put a large tinfoil dish at the L2 point behind it to focus the sunlight, and the chunk of ice and rock turns into a mini planet with a warm water ocean, maybe some ammonia. Great, except... no air, only some water vapor. So you fenestrate your membrane with little holes that let hydrogen seep through freely, and do anything you can to make the membrane kinetically catalyze the equilibrium 2H2+O2 <-->>>>> 2H2O and the same for ammonia (but easier for that, I should think). Logically, the hydrogen should go out through the holes and never be seen again.

Now, I can read from electrolysis of water that it takes "286 kJ of electrical energy input to dissociate each mole", i.e. 16 kJ per gram of water (MW 18). Work done by pushing water through a pressure differential should be delta P * V, i.e. for one cc (gram) of boiling water (1 atm = 100 kPa to 0 atm) it would be 100000 kg/ms^2 * 0.000001 m^3 = 0.1 kgm^2/s^2 = 0.1 J. Harrumph. If I'm right, that's amazingly disappointing!

Yet my feeling is that it should happen anyway - that there should be some equilibrium value for hydrogen pressure outside the membrane. If it were encapsulated by a second membrane, then it would build up to some miniscule value and eagerly react and reenter. But it's not - the moment one molecule of H2 gets free it should head for deep space and never be heard of again. Which is weird - it's as if I expect some additional work to be done on the water by "extracting" the lack of entropy from deep space.

Where am I going wrong here? And, yes, can we actually make comfy little planets this way? Wnt (talk) 17:31, 11 February 2013 (UTC)[reply]

Wouldn't it get colder? You are essentially allowing a gas to expand which will get colder, and eventually stop the hydrogen escaping. The sun heats it up though, so that's your energy source. Ariel. (talk) 18:56, 11 February 2013 (UTC)[reply]
Hmmm, I was thinking the hydrogen could get cold "out in space" and not affect the inside of the membrane, at least conceptually, though I don't actually know if that is possible even in theory. On second thought though, I suppose that the "very rare molecules" that would somehow manage to spontaneously split in equilibrium must be at the very highest energies present in the Boltzmann distribution - and that kinetic energy would be the source of the energy to break the chemical bonds. Once removed, the liquid as a whole gets cooler. Yes, I think that might just be the answer - I'm actually using the heat, not the pressure, to split the water. Wnt (talk) 05:28, 12 February 2013 (UTC)[reply]
Can you elaborate on your calculation? As stated it makes no sense to me. What are you calculating? Dauto (talk) 21:20, 11 February 2013 (UTC)[reply]
I was calculating the amount of energy needed to split a gram of water, versus the amount of energy that should be produced if the gram = cc of water is pushed out under pressure. True, only the hydrogen actually is supposed to be pushed out ... but it was such a low number I didn't try to figure out that detail. I'm not sure it's the right way to do that calculation anyway... Wnt (talk) 05:24, 12 February 2013 (UTC)[reply]

tube amplifers

looking for information on how to build a simple tube amplifer — Preceding unsigned comment added by 98.235.151.172 (talk) 18:12, 11 February 2013 (UTC)[reply]

D-d-d-danger! High Voltage! [19][20][21] don't come crying to us when you electrocute yourself to death ;) ---- nonsense ferret 18:27, 11 February 2013 (UTC)[reply]
I suggest you google it or visit your local library. Ratbone 120.145.64.230 (talk) 01:21, 12 February 2013 (UTC)[reply]
The "Bear Scout" handbook (US Cub Scouts) in the 1950's had a schematic for a simple 1 vacuum tube radio for children to build, which used a tube intended for portable radios powered by dry cell batteries. I doubt that it used a very high plate voltage. I have used several 9 volt batteries connected in series for the plate supply to run an amplifier using early tubes such as the UX201A . If I recall correctly it was designed to run on a 90 volt plate supply but would also work as low as 45 volts. Note that even these voltages can cause dangerous shock. Some tube will provide some amplification down to 22 volts of plate voltage, though the amplification may be low. Most schematics on the internet seem to be for "modern" 1960's high power-high fidelity amplifiers, with very high plate voltages, Libraries might have "Radio physics course" by Alfred Ghirardi (1933) which includes diagrams for simple low power low fidelity inefficient amplifiers with early tube types (which can be bought on Ebay). Edison (talk) 04:20, 12 February 2013 (UTC)[reply]
Depending on your focus; if you are more interested in the amp part than the building part (i.e. if you just want to try "tube sound" or something) the easy way is to get an old tube type radio and feed signal in to the volume control, via a capacitor. Gzuckier (talk) 06:19, 12 February 2013 (UTC)[reply]
Ah the good old days when I converted a television into a monitor with sound and without the need for the UHF stage. Seriously though even when you switch off and unlug the power you can get a nasty shock because of capacitors, if you have to fiddle with it while switched on make sure you keep one hand behind your back and keep insulated - and don't blame us if you use high voltages and kill yourself. BTW a concrete floor can conduct, use rubber shoes or a rubber mat to stand on. Dmcq (talk) 10:23, 12 February 2013 (UTC)[reply]
And of course those ubiquitous 5 tube table radios had the chassis connected directly to one side of the AC plug; and zillions of them were made for decades before the polarized AC plug/socket appeared (in the US anyway). Well, that's a 50/50 chance, anyway.Gzuckier (talk) 15:50, 12 February 2013 (UTC)[reply]
If you Google "tube amplifier kit" there are lots of them available for a few hundred dollars. To build a basic 1 tube audio amp circuit as a demonstration, the 1T4 pentode is popular, with a 1.5 volt filament and a typical plate voltage of 45 volts (still a dangerous voltage, so caution is required). The tube itself is readily available for a few dollars. Googling "1t4 tube amplifier" will show many example circuits. Edison (talk) 15:14, 12 February 2013 (UTC)[reply]

February 12

Engineering contractors

I sometimes hear of "Engineering contractors" and "Engineering consultancies". Is an Engineering contractor in CIvil Engineering basically a construction company? Clover345 (talk) 00:15, 12 February 2013 (UTC)[reply]

This may vary from country to country. However, in the USA and in Australia, and engineering contractor would normally be a firm subcontracted to a construction contractor acting as prime contractor (i.e., the firm that is hired by the customer that wants the thing built), to provide engineering services to the prime contractor. These engineering services will include design calculations and production of detail drawings (as distinct from concept drawings produced by the customer or architect) as well as installation of engineering products such as pipes, electrical services, etc. An engineering consultant is generally a firm or individual that specifically concentrates on calculations and drawings. Doing consultancy work has connotations of doing the less routine or more specialised calculations. For example, if during the construction of a large hydro-electric dam, the engineering contractor's staff notices that subsidence is greater than expected from their own calculations, they may call in a consultant to advise them on what to do about it, if anything.
A firm of consultants will typically have a higher percentage of staff with considerable experience and advanced knowlege, and university degrees are essential (and may comprise of just the one owner /engineer), whereas an engineering contractor will have a wide range of staff with all sorts of trade and professional qualifications, and some will have less experience; some will have just graduated.
An engineering contractor's reputation and ability to get work depends on his ability and reputation to get things done on time and within budget. A consultant's reputaion and ability to attract work depends more on his ability to solve novel problems and get the right answer.
There is considerable overlap - a firm of consulatnts may often accept work that involves straightforward calculations and may go on site to inspect or supervise work.
Ratbone 01:47, 12 February 2013 (UTC) — Preceding unsigned comment added by 120.145.64.230 (talk)
I know someone whose title is "project engineer" who works for an engineering consultancy firm. His firm is hired by customers to oversee the project as a whole, to make sure that the architect's plans are built according to spec and within code and on time and on budget by whatever various construction firms do the physical works. His main task, besides oversight of the work, is scheduling, since time is money, but certain work has to be done in a certain order. He deals with architects whose plans are impossible to build, either due to cost or bad design, construction companies that pad the budget and cut corners, and customers who want the impossible for less money and in less time than is required. Besides building nuclear plants, high schools, bridges in NYC, and luxury high-rises overseas he spends much of his time in court testifying when the owners, architects, and construction companies sue each other. μηδείς (talk) 02:05, 12 February 2013 (UTC)[reply]
You are more of a cynic than I, Medeis, but you are not wrong. You have reminded me that that consultants get hired to settle disputes - either by the customer who thinks he's been taken down, or the contractor who thinks he should be allowed to make a profit, or by someone's legal team who need an expert witness in court to bamboozle the judge. Ratbone 120.145.64.230 (talk) 02:18, 12 February 2013 (UTC)[reply]
Well, I am just repeating stories I have heard. But googling the name of the gentleman I was talking about you'd think he was a major criminal for the number of lawsuits he's named in--yet his reputation is such people fight over him when he's available for the next project. One other point. He's not normally the main contractor as in the agent who hires the subcontractors. Usually the owners have already taken bids or have usual agencies they work with. He can recommend them if that's what the owner wants, but he doesn't pay them, and he is only paid by the owner, or it would be a possible conflict of interest. μηδείς (talk) 02:42, 12 February 2013 (UTC)[reply]
Building off Medeis' answer, note also that "engineer" can be a regulated professional title like "lawyer" or "doctor", see professional engineer. I would expect that an engineering consultation firm, particularly in civil or mechanical contexts, is centered around one or more PEs who are legally able to sign off on the stuff that needs to be signed off -- that would certainly also explain "spending a lot of time in court testifying". Of course, engineering is a little weird when contrasted with lawyers and doctors in that most engineers who are professionally employed as engineers are nonetheless not professional engineers. I am one such, partly because I don't need it for my job and partly because the exams don't map well to my discipline (and so it wasn't worth the effort back when it was convenient, much less now). — Lomn 13:41, 12 February 2013 (UTC)[reply]
Agree with all that. Some civil engineers avoid architects like the plague and just do the design themselves, though some architects do have a grasp of realities and they can produce very good designs. And yes it is amazing how much time an engineer in management can spend in court even sometimes fighting the government over silly things it tries to do or contracts it tries to get out of. Dmcq (talk) 10:34, 13 February 2013 (UTC)[reply]

Densest element ?

what is a solid element with the greatest density — Preceding unsigned comment added by 98.127.57.58 (talk) 01:06, 12 February 2013 (UTC)[reply]

I added a title. StuRat (talk) 01:08, 12 February 2013 (UTC)[reply]
Note that elements aren't solids, liquids, or gasses, per se, but exhibit different states at different temperatures and pressures. I will assume you mean chemical elements which are solid at room temperature and standard atmospheric pressure. StuRat (talk) 01:11, 12 February 2013 (UTC)[reply]
Standard temperature and pressure is probably a good assumption for a common point of measurement, but I doubt that we actually need to artificially limit things to solids -- they're likely to be more dense than gases/liquids anyway. Here is a list of elements by density at STP. That site further notes that there is some uncertainty regarding calculated densest vs experimentally measured densest, noting that while iridium is theoretically denser than osmium, that has not yet been experimentally measured. We list osmium as "the densest naturally occurring element" (emphasis mine). Some transuranic elements quite probably are more dense, but that likewise has not yet been definitively verified. — Lomn 01:23, 12 February 2013 (UTC)[reply]
I added a redirect for densest element, but already there was a redirect for the densest element. Graeme Bartlett (talk) 20:43, 12 February 2013 (UTC)[reply]
I'll be honest; I'm not thrilled by pointing those redirects straight to osmium. They declare (in the encyclopedia's voice) a single answer—one which likely won't be true in all temperature and pressure regimes, and (as Lomn notes), isn't necessarily certain even at STP. Better targets might be the List of elements or even our article on Density (which mentions both osmium and iridium jointly as the densest elements)...if we actually really want those redirects at all. TenOfAllTrades(talk) 17:28, 13 February 2013 (UTC)[reply]

My old socks said they had elastic in them, but the new ones say Spandex, instead. Are these two different names for the same thing, or are they different ? If different, will Spandex last as long ? I use bleach in the wash, so how well do both hold up to that ? StuRat (talk) 01:49, 12 February 2013 (UTC)[reply]

Stu, you should know better than most to just look up our Wiki article on Spandex first. It will answer your first question at least. Googling will answer the rest. Ratbone 120.145.64.230 (talk) 01:53, 12 February 2013 (UTC)[reply]
That tells me what Spandex is, but not what "elastic" is. It's apparently a catch-all term for a variety of elastomers, possibly including Spandex. So, I'm asking which of those is likely to be used in socks. StuRat (talk) 02:02, 12 February 2013 (UTC)[reply]
I am astounded that we don't have an article or redirect for the variety of elastic that I am familiar with, which is used in haberdashery. I believe what would be used in socks is known as "shirring elastic", rolls of which are pictured here. It's basically something the thickness of thick sewing thread or thin knitting yarn, but stretchy. --TammyMoet (talk) 09:45, 12 February 2013 (UTC)[reply]
Thanks. Do you know how it compares with Spandex, as far as longevity and, specifically, ability to withstand bleach ? StuRat (talk) 17:02, 12 February 2013 (UTC)[reply]
I always thought Spandex was an elastic fabric rather than a fabric with elastic woven or knitted into it. As far as ability to withstand bleach goes, I would say it fails on that count, although I can't say I've ever bleached elastic. I'll have a look at my household hints stuff and get back to you. --TammyMoet (talk) 20:42, 12 February 2013 (UTC) http://housekeeping.about.com/od/laundry/qt/bleachtips.htm says don't bleach Spandex or elastic. (Why would you put bleach in a normal wash anyway?) --TammyMoet (talk) 20:44, 12 February 2013 (UTC)[reply]
Yes, I am fairly confused by this. Are the entire socks spandex, or just their collars? μηδείς (talk) 20:58, 12 February 2013 (UTC)[reply]

Media coverage of coronaviruses compared to other infectious diseases

Why do coronaviruses get so much media attention compared to other infectious diseases, when they affect far fewer people? As as I write this, many news organizations have novel coronavirus 2012 as one of their front page headlines (specifically regarding the recent discovery of a case in England). And I remember how heavy the news coverage of SARS was back in 2003. Going by the statistics on their corresponding articles, malaria kills about 1 million people a year, and influenza kills a 6-digit number of people a year. SARS killed around 800 people, and only a handful of cases of the novel coronavirus have been identified. In the case of malaria, it is a disease affecting mostly developing countries, so that could be an explanation for why media organizations in developed countries would not pay as much attention, but influenza, on the other hand, is widespread in developed countries, and yet these coronaviruses are getting just as much or more press attention despite affecting far fewer people. Is this just a case of media sensationalism, or is the threat of coronaviruses actually high enough to justify the density of their media coverage in comparison to other infectious diseases? —SeekingAnswers (reply) 05:48, 12 February 2013 (UTC)[reply]

SARS had a death rate of about 10%, which is pretty freaking scary all on its own. If there's less total deaths it could in part be because of the attention paid to such infections, which helps restrict transmission and keep deaths low. I'd hate to see a SARS outbreak where as many people were infected as typically get infected by the flu, and if the current level of attention to coronaviruses helps keep the total deaths low, I can't see that as a bad thing. Influenza may kill more total people every year than coronaviruses, but given the total number of infections, it appears to be a much less serious infection to get. The most generous figures for the 1918 flu pandemic gives numbers that approach the 10% kill rate of SARS, though numbers closer to 2-5% seem more reasonable, and that's widely recognized as the worst flu outbreak in history. The recent 2009 flu pandemic (the swine flu/H1N1 outbreak) was considered considerably bad, and using confirmed cases the Wikipedia article cites a kill rate of 0.03% in a chart near the bottom; the lead of that article notes that 11-21% of the world population was infected and gives an upper limit of 579,000 or so deaths world wide. That means that, if we take a worst case scenario in terms of calculating deadliness, using the lower infection rate and upper death numbers, we get 11% of 7 billion = 770,000,000 infections, and 579,000 deaths is 0.08% of infected patients that died. If a SARS-like coronavirus had a similar rate of infection, it would have caused 77,000,000 deaths. For comparison, the 2009 Flu pandemic killed the population of Las Vegas, Nevada. A similar SARS outbreak would have killed the combined populations of California, Texas, and Illinois. So, I'd say the fears are not unfounded. --Jayron32 06:08, 12 February 2013 (UTC)[reply]
Agreed. This also feeds into human psychology. That is, we've evolved to be more interested in new threats than ones we already know about. In that way, we were able to develop ways to defend ourselves from new threats. However, broadcast media tends to exaggerate the threat for ratings, which makes us even more interested, thinking it really is a serious threat to our survival, when there's really only a very low chance it ever will be. So, the mass media doesn't help us improve our health, when they harp on obscure threats, instead of things which are more likely to kill us, like alcohol, tobacco, bad diets, and a lack of exercise. StuRat (talk) 06:22, 12 February 2013 (UTC)[reply]
Stu, I'm gonna have to say that theory is deserving of a big fat [citation needed]; I know of no serious researcher within the field of evolutionary psychology who has posited, let alone tried to support, that we are innately conditioned to be more fearful of threats which are new to us as opposed to known dangers. There's really no way to control for or test such a prejudice and no empirical reason to assume it in the first place; there are plenty of counter examples (phobias developed early in life, for example, can often be the most gripping) and in any event quantifying levels of of fear methodologically is difficult, bordering on impossible (there are plenty of studies involving various forms of brainscan which center around fear, but none of this gives us an unambiguous metric). Even establishing what constitutes a "new" threat across the perspectives of multiple individuals is rife with complications. Across populations, we can recognize trends as to what people pay attention to, especially with regards to popular media, but, as everyone who has commented here has already noted, there are any number of social explanations for those trends which do not necessarily reflect an absolute correlation to the relative level of fear/panic/what-have-you that people have regarding disparate potential threats. The examples you give of other dangers which we are prone to ignore (tobacco, alcohol, unhealthy food and lethargy) are also all examples of things which can be readily explained (in part at least) by the pleasure, comfort, and dependency issues involved. Aside from which, your assumption excludes the fact that there are plenty of examples of potential pathogens and dangers which are new which do not become lightning rods for levels of fear that are out of proportion to older dangers which are "greater". For that matter, establishing just what is in reality a greater threat is often going to be a matter of intense debate.
All of which is not to say that people cannot become desensitized to well-known dangers (that's a simple intuitive assumption concerning human psychology which I assume we can all agree on) and there may even be a seed of truth to your theory (albeit with a thousand caveats), but it's highly inaccurate to present this as an established psychological feature that is the result of a discreet evolutionary adaptation towards novelty -- and the reality of how people process fear and adapt to new threats is quite a bit more complex than such an argument would suggest. Snow (talk) 01:04, 13 February 2013 (UTC)[reply]
Here's a studt talking about the process whereby old fears lose their impact on us: [22]. We also have our own article: Extinction (psychology). If you don't think this is an evolutionary adaptation, why exactly do you think fears of old threats fade relative to fears of new threats ? StuRat (talk) 04:58, 13 February 2013 (UTC)[reply]
You've completely misrepresented that study. Here is the very first sentence from the abstract: "Conditioned fear responses to a tone previously paired with a shock diminish if the tone is repeatedly presented without the shock, a process known as extinction." Not even remotely what you are proposing.
As to your question, let me first remind you that my above post includes this: "All of which is not to say that people cannot become desensitized to well-known dangers (that's a simple intuitive assumption concerning human psychology which I assume we can all agree on)". But your statement that fears of old threats always (or even generally) fade relative to fears of new threats is pretty radical assumption to just take for granted. For plenty of people the greatest and most crippling fears they have are ones picked up many years earlier; consider victims of abuse or sufferers of PTSD or those with intense phobias. A lot of interconnected factors go into determining an individual's fear response; is the fear an innate one (most people will react with fear or startlement to a loud noise in close proximity) or is it conditioned? If acquired, how traumatic was the precipitating event and over what period did exposure occur? If truly traumatic, do they receive assistance in coping with the fear? How rational or irrational is the fear and thus how much can it be mitigated by understanding or prevention/avoidance? If less a case of a trauma and more of one in which a person is exposed to a danger only in abstract, what is the context in which they learn about it and how fallible (or infallible) do they find the source informing upon this threat to be and how much contrary evidence are they exposed to afterwards? At what age was the fear acquired? Do the perspectives of those around a person reinforce or run contrary to this fear? These are just a few of the myriad factors which could influence how long a given "threat", as you put it, could be considered particularly dangerous by a particular individual.
The amount of time passed since the person was first exposed to a given fear stimuli is of relevance, of course, but you're not even suggesting just that -- you're proposing that novel concepts are more likely to instill more fear relative to old ones and I just can't see how you'd ever go about proving that (or why you'd assume it). This belief seems to be based on other subordinate false assumptions, namely that fear is rational and that it makes sense as an adaptation in this context. It doesn't though, really. You see, our fear response, like most of our psychological make-up, was created well in advance of the kind of threat one tackles with abstract thought; a strong fear response has value as an evolutionary adaptation when you are talking about fight-or-flight situations, especially with the manner it's tied into the autonomic nervous system, but its usefulness is far more questionable when you're talking about prioritizing and analyzing complex concepts like the relative threats posed by two different diseases, the context in which this discussion arose. These just aren't concepts which hunter-gatherers or our animal predecessors were coming to grips with during the vast, vast majority of the evolution of our neurological mechanisms for processing fear; these two different types of danger --obvious immediate visceral threats and long-term abstract ones-- are even processed by differing brain regions with only a certain amount of overlap. But even if these were fears that we had struggled with for more of our evolutionary history, how would it be beneficial for the human mind (or any thinking entity) to assume a new abstract threat is more dangerous than an old one? There's no reason to assume that if you're going to be stricken by a disease it will be a new one so what evolutionary pressure would there be create this bias as an innate constant in the human mind? Snow (talk) 08:10, 13 February 2013 (UTC)[reply]
News organizations report what's new. Influenza and malaria have existed for the entirety of human history, and probably much longer than that. Any educated person, and most uneducated people, already know about them. --140.180.243.51 (talk) 06:59, 12 February 2013 (UTC)[reply]

Identify a part of the body

Right above the sternum where a couple of muscles form a sort of V shape, there is a depression in the neck. On men it is just below the Adam's apple. What is this depression called, if anything? I've been watching House of Cards and it's very pronounced on Robin Wright. Dismas|(talk) 09:52, 12 February 2013 (UTC)[reply]

It is the suprasternal notch, also known as the jugular notch. Gandalf61 (talk) 10:12, 12 February 2013 (UTC)[reply]
Thank you. Dismas|(talk) 11:03, 12 February 2013 (UTC)[reply]
That doesn't sound right. I am fairly certain its the ucipital mapilary. μηδείς (talk) 18:20, 12 February 2013 (UTC)[reply]
Did you click on the link you provided, Medeis? 86.163.209.18 (talk) 19:24, 12 February 2013 (UTC)[reply]
I think that was supposed to be a joke. Sigh. Looie496 (talk) 22:13, 12 February 2013 (UTC)[reply]
But in this case, I think Medeis has underestimated the obscurity of her reference. Snow (talk) 22:41, 12 February 2013 (UTC)[reply]
In terms of a fact, it is simply wrong, and unhelpful left unmarked on the desk. I considered asking Medeis if it was a joke, but my experience is that people get cross if you ask that and it was simply ignorance. If I ask whether they checked the link, and it was a joke, she gets to feel quietly superior and assume that I am stupid. This is less disruptive to the desks, since I don't care if people think I'm stupid. 86.163.209.18 (talk) 23:05, 12 February 2013 (UTC)[reply]
Well it would be pretty silly if someone assumed you stupid for knowing the actual clinical name of an anatomical feature but not an obscure fictitious variant. In any event, I don't think Medeis intended any harm with the remark, regardless of motive, and the clarification needed to understand the joke for what it is (if it was indeed a joke) is found within the article cited, so I don't think anybody was likely to be misled or confused for long. Snow (talk) 00:10, 13 February 2013 (UTC)[reply]
I knew it was a joke phrase the first time I heard Cary Grant say it--it's obvious mock Latin--I just didn't know there was an actual name for the anatomical feature until now. You may notice I waited for the right answer to be posted and Dismas to acknowledge it (presumably having read the article) before I posted my allusion, and lank again to the same article. μηδείς (talk) 01:53, 13 February 2013 (UTC)[reply]
So if you were joking, why no <small> tag? Absent that, it was just misleading information that could easily confuse the OP. Don't make jokes like this - few people find them amusing and if there is even the tiniest chance of confusing the OP, they are a monumentally bad idea...and as you are aware, we confine jokes to the small font. SteveBaker (talk) 15:14, 13 February 2013 (UTC)[reply]

For what it's worth, I did read the article and the joke was completely lost on me. Dismas|(talk) 03:58, 13 February 2013 (UTC)[reply]

Protocol for asteroids

Who is in charge exactly (I suppose the NASA) for monitoring asteroids hitting the Earth and how would they communicate a potential danger to the public? (if at all). OsmanRF34 (talk) 14:30, 12 February 2013 (UTC)[reply]

See Near-Earth Asteroid Tracking and links from there. Torino Scale and Palermo Technical Impact Hazard Scale also have some relevant information. --Jayron32 14:46, 12 February 2013 (UTC)[reply]
NASA generally issues press releases or conducts press conferences for major announcements. --PlanetEditor (talk) 14:55, 12 February 2013 (UTC)[reply]
Also chance of an asteroid hitting earth and causing massive damage in the planetary scale is miniscule in the next 1000 years.. However if a timescale comprising millions of years is considered, there are good chances of future impact events which occurred thoughout Earth's history. --PlanetEditor (talk) 14:59, 12 February 2013 (UTC)[reply]
Note that a portion of the Southern sky around the South Celestial Pole is currently only being monitored by this guy (see last paragraph of article), and he's just had his NASA funding cut, so until a new project cuts in a couple of years hence, while Rob's on holiday no-one is looking out for the incoming from that part of the sky. (Rob was a classmate of mine at Uni, so I follow his career somewhat – it's the one I failed to attain!) {The poster formerly known as 87.81.230.195} 84.21.143.150 (talk) 16:27, 12 February 2013 (UTC)[reply]
The Minor Planet Center collects worldwide reports for all small objects. However they do not make observations themselves but collect them from amateurs and professional (mainly computers nowadays). There are several different near earth object detection programs - none is anywhere near comprehensive. Asteroid impact avoidance mentions some ongoing projects but some of those have already ended. Rmhermen (talk) 18:55, 12 February 2013 (UTC)[reply]

Camphor wood

is it safe to work with the wood from a Camphor tree?

Effects on the body

It has effects similar to a muscarinic receptor agonist.

[edit] Small dose

Its effects on the body include tachycardia, vasodilation in skin (flushing), slower breathing, reduced appetite, increased secretions and excretions such as perspiration, diuretic. [16]

[edit] Large dose / toxicity

Camphor is poisonous. In large quantities, it produces symptoms of irritability, disorientation, lethargy, muscle spasms, vomiting, abdominal cramps, convulsions, seizures.[17][18][19] Lethal doses in adults are in the range 50–500 mg/kg (orally). Generally, two grams cause serious toxicity and four grams are potentially lethal.[20] — Preceding unsigned comment added by 70.119.184.168 (talk) 17:38, 12 February 2013 (UTC)[reply]

It seems to be a sought-after wood for woodworking[23]. Many species of exotic wood have toxic dust or oil that require careful handling though they are still used for woodworking. Camphor is not listed on the list of toxic woods here[24] (but sneezewood is!) Rmhermen (talk) 19:16, 12 February 2013 (UTC)[reply]
By and large you don't want to inhale large quantities of sawdust on a regular basis in general, IIRC. Gzuckier (talk) 05:20, 13 February 2013 (UTC)[reply]

Why was Typhoid Mary an asymptomatic carrier?

I read Typhoid Mary and Asymptomatic carrier, but I didn't see what physiological/genetic attribute is posited to account for her unaffectedness. Did her immune system luckily happen to differ such that it naturally knew how to combat the pathogen? What was it about her? What does it tend to be that saves asymptomatic carriers of infectious diseases? 20.137.2.50 (talk) 19:13, 12 February 2013 (UTC)[reply]

Like many infectious diseases, people who get the disease once may develop a lifelong immunity to it due to the build up of antigens in their system. These antigens can protect them from coming down with symptoms of the disease, but do not necessarily prevent them from carrying the disease around on the clothes, hands, or even internally. There's a possibility that Mary had a mild case of Typhoid fever when younger, which had been written off as a bad cold or other relatively mild infection, and from that point forward she became a carrier. --Jayron32 19:47, 12 February 2013 (UTC)[reply]
Does anybody but me find that amazing? Gzuckier (talk) 05:21, 13 February 2013 (UTC)[reply]

The article above talks about Gibbs in a thermodynamics context but what is its significance in Biology? Clover345 (talk) 19:52, 12 February 2013 (UTC)[reply]

Biology is not immune to thermodynamics. Biological processes still obey all of the basic laws of the universe. --Jayron32 19:55, 12 February 2013 (UTC)[reply]


Yes, there is explicit use for Gibbs free energy (GFE) in biology, for example this paper uses it quite a bit: "Effect of classic methanogenic inhibitors on the quantity and diversity of archaeal community and the reductive homoacetogenic activity during the process of anaerobic sludge digestion" here ([26]). This usage is in line with Jayron's comment above. However, there is another case that may come up, that is more in the context of analogy, due to the same mathematical structures coming up in different applications. For instance, this work ([27]) uses methods from statistical mechanics to analyze trophic cascades, and concludes in part that GFE gives the netproductivity of the ecosystem. Another lead is that shannon diversity is essentially a measure of entropy, and it is widely used in ecology (not because of thermodynamics, but because it is a useful way to quantify diversity). I can't recall the appropriate physics, but there may be some related applications of GFE, due to the relations between entropy and enthalpy. Finally, you may be interested in this review of thermodynamic processes in ecology [28]. SemanticMantis (talk) 21:22, 12 February 2013 (UTC)[reply]

Properties of blood

From my understanding, blood is a shear thinning fluid like custard and so this would contribute to blood clotting when it's flow velocity is reduced such as in the case of bleeds. But I also understand that clots and recovery from bleeds are to do with platelets, oxidisation of blood etc. So how do these 2 concepts come together when a bleed and subsequent recovery occurs? Clover345 (talk) 19:57, 12 February 2013 (UTC)[reply]

Thixotropic fluids are thixotropic because they are suspensions and not really pure liquids; the suspended particles provide the shear thinning property. It is specifically because of things like platelets suspended in the blood that would give it such a property. --Jayron32 20:39, 12 February 2013 (UTC)[reply]
So in other words shear thinning properties are just a way of modeling fluids? For example, would the fact that custard is a shear thinning fluid also be because of suspensions? Clover345 (talk) 22:00, 12 February 2013 (UTC)[reply]
Sheer thinning/thixotropy is a real property: Some fluids to decrease their viscosity at higher flow rates. And what do you think custard is if it isn't a suspension? --Jayron32 22:03, 12 February 2013 (UTC)[reply]
I don't know. Suspensions of fat in milk, egg yolks and cream? But Hemostasis and Platelet doesn't mention anything about shear thinning properties and this confuses me. Clover345 (talk) 22:06, 12 February 2013 (UTC)[reply]
A suspension just means the distribution of aggregate particles within a liquid medium; that is it is distinct from a solution in that the solution is composed of molecule-sized particles distributed in the liquid phase; while in a suspension the particles are clumps of molecules, often many millions or billions of them. In custard, your liquid is water, while the suspended particles are the fats and proteins from the cream and eggs. The lecithin from the eggs acts as an emulsifier which keeps the particles suspended and prevents them from settling out. If you want to know more about thixotropy or sheer thinning in blood, see this Google search. If you find relevant information in that search which isn't already in Wikipedia article, but belongs there, be our guest and add it. --Jayron32 22:15, 12 February 2013 (UTC)[reply]
Ive heard platelets work better at lower velocities. From my understanding, platelets work by plugging the wound along with various proteins that aid in the wound recovery process. Ive found out from here (http://www.cs.cmu.edu/~sangria/publications/MAKRR2004.pdf) that the plasma of blood itself is a Newtonian Fluid. The article also states "In the future, we intend to incorporate the

process of clot formation due to platelet activation in blood flow, the Schematic of the natural configurations associated with a clot being modeled as a viscoelastic fluid or solid (there is some viscoelastic fluid having a single relaxation mechanism, and debate on this issue)." So does anyone actually know yet how the blood's shear thinning fluid properties and the biochemical processes interact in the wound healing process? — Preceding unsigned comment added by Clover345 (talkcontribs) 00:00, 13 February 2013 (UTC)[reply]

high school physics

assume instead of a large battery an rc plane is carrying a parachute with solar panels and a rather small battery (maybe even motor capacitor). The question is: could the parachute have enough solar material on it that by the time the rc plane has finished falling from high altitude it can recharge enough to climb to the same altitude? (Assume it packs away its 'chute again somehow). This is a high school physics question and has no relevance to anything. Boriskol (talk) 20:46, 12 February 2013 (UTC)[reply]

High school physics questions aren't supposed to be relevant. They're supposed to get you to learn to pick the correct equation from a scenario and to assign the correct numbers from the problem to the proper variables in the equation. The idea is to train you to understand how mathematics and the physical world interface, and how to recognize what mathematics to do when faced with any sort of problem. But the first training that a student needs to do is simply read the description of a scenario, and then translate that into math and produce a numerical result. Once you have that practice down and can do it easily, then it becomes easier to progress to more difficult concepts. --Jayron32 20:50, 12 February 2013 (UTC)[reply]
This is a single use account with a sophisticated knowledge of editting WP to ask a question that "has no relevance to anything". Closure seems reasonable. μηδείς (talk) 20:55, 12 February 2013 (UTC)[reply]
I disagree. --Jayron32 21:17, 12 February 2013 (UTC)[reply]
Medeis, as has been explained to you before, your previous attempts to close down conversations have been the cause of significant conflict. I strongly advise you to cease all activity involving closing, collapsing, or deleting any other editor's comments. The reference desks have plenty of eyes on them. if something needs to be closed someone will close it. At this point your even suggesting that something be closed is likely to cause a backlash in favor of keeping it open. While no policy says that you cannot make suggestions, everything would go a lot smoother if you would just stick to making your own comments and stop trying to control other editors in any way. See WP:OWNERSHIP. --Guy Macon (talk) 06:21, 13 February 2013 (UTC)[reply]
(ec)Let's not jump on a first-time OP simply because he was being modest about the importance of his question. It's an interesting question, and I for one would like to see an answer. Since this is a reference desk, maybe someone knows of a reference that discusses a successful or unsuccessful attempt to do this or something similar. Duoduoduo (talk) 21:24, 12 February 2013 (UTC)[reply]
Is this User:91.120.48.242 again? If so, User:Totallyabstract is a sock-puppet...which is A VERY BAD THING. Look, your Helium-balloon electric R/C plane doesn't work, neither does hot-air-balloon plane - and neither will the parachute-plane. No matter what you do, in any wind over a couple of mph, and with the most optimistic assumptions, for any conceivable balloon/parachute size, solar power coverage, battery pack capacity and engine efficiency - the balloon/parachute/whatever will get blown further off-course vastly faster than the plane can make up with whatever charge can be stored. I can re-quote the math I gave you via email if you'd like. <sigh> SteveBaker (talk) 21:57, 12 February 2013 (UTC)[reply]
"Sophisticated knowledge of editting (sic) WP" Ha, good one! This desk is literally 2 clicks away from the main page; apparently that counts as "sophisticated editting". --140.180.243.51 (talk) 21:25, 12 February 2013 (UTC)[reply]
That's just one of many brilliant contributions of μηδείς. OsmanRF34 (talk) 21:51, 12 February 2013 (UTC)[reply]
What is the area of the parachute? What's the mass of the plane? What's the terminal velocity of the parachute + plane combination? Are you given any parameters at all, or are you supposed to choose reasonable ones? --140.180.243.51 (talk) 21:25, 12 February 2013 (UTC)[reply]
Given that there are very real solar-powered planes that can fly indefinitely, the answer is absolutely "yes". This isn't a perpetual motion scenario because you've got energy input via the sun. — Lomn 21:28, 12 February 2013 (UTC)[reply]
Yes the scenario immediately seems excessively convoluted when the plane's wings could just be covered with solar panel in the 1st place.. Vespine (talk) 21:42, 12 February 2013 (UTC)[reply]

Ah, where I said "high school physics question" I meant question utilizing high school physics. The question is of my own devising. I simply meant that this has no practical applications whatsoever. The main difference, I suppose, between "just putting solar panels" on the wings is that you would then have a different plane: a typical RC plane draws more power than you would get by putting solar paneling on, so you would have to build something different.

So, the question is about the physics of an RC plane, wherein the batteries are assumed to be mostly replaced with the rest of this setup. The question is: would a large parachute, with solar paneling, provide enough sustained power over the course of descent, for the RC - if we assume it can efficiently pack that 'chute back up - to climb just as high as it had been? I am not sure what parameters are reasonable here as I have no direct experience with this. For me, it's just a high school physics type thought experiment with no applications of any kind. I was hoping some of you would know how to calculate the answer. Boriskol (talk) 22:04, 12 February 2013 (UTC)[reply]

re: "you would have a different plane" -- well, that's going to be the case regardless. A solar-powered aircraft will of necessity have different design considerations than a conventionally-fuelled aircraft, regardless of scale. That will hold whether you put solar panels on the wings, on a parachute, or on a big kite you tow behind the main aircraft. So, that said, the answer still remains that it is scientifically possible -- as noted, you've got energy input via the sun. As for how you would actually do this -- whether it's practical from an engineering standpoint -- you quickly move past high school physics. What's the terminal velocity of the aircraft while parachuting? That's how long you've got to charge. What's the maximum charge rate of your solar panels? There's your energy budget. What's the mass and drag of the aircraft under normal flight? That's whether your energy budget is sufficient. You can pick answers for some of those and, with high school physics, derive approximate requirements for the others. However, it won't answer the practical questions of whether your design will work; for anything reasonably cutting-edge, that's better considered as aerospace engineering -- and that, I think, is the distinction Jayron intended with his initial response. — Lomn 22:21, 12 February 2013 (UTC)[reply]
Indeed. High school-level physics (as with all high-school level courses, in nearly all disciplines) is about teaching you the language and methods of studying that discipline. Physics is no different. Most of high school physics is intuitive from a non-mathematical point of view; to the point of being nearly obvious. If you push something harder, it goes faster. If you're going faster, you get where you were going in less time. Stuff like that. The first goal of such courses isn't so much to teach you the material (I mean, do you need such a class to teach you that if you go faster, it takes less time to get places?). Instead, it's to teach you how to think critically about a situation, to learn how to apply mathematics to situations you describe in words, to learn how equations work, to learn how to solve such equations, what the definitions of words are. There's some small amount of actual material you are learning, but such material is actually secondary to the methods: Once you know how to solve such problems, you can pretty much teach yourself the material. --Jayron32 23:19, 12 February 2013 (UTC)[reply]
yeah, the parachute part of the problem is sort of a distraction. If you simplify the situation, the existence of solar powered planes which can fly contiually (which would be more easy with no pilot than with, I would guess) sort of represents the limiting case, where the run out of juice and parachute phase of the trip represents zero percent. Of course you would have to build a very energy-efficient plane, which resembled the solar powered model as much as possible. The parachute itself is just a complication; in addition to excess weight that has to be lifted, it seems when deployed it would be likely to shade the cells. I don't know of any cells that would function on top of a parachute very well. In fact, anything that efficient would probably be just as good at recharging while gliding. (I should point out that I don't know anything about the subject this is all off the top of my head) If you are married to NOT having the solar cells inhabit the surface of the plane, however, I think that approach is a dead end. Gzuckier (talk) 05:31, 13 February 2013 (UTC)[reply]

Little Ice Age

Another entry under 'causes' might be useful, to give a full picture.

The simplest cause of the Little Ice Age - and all other incidental cold spells - is our Solar System passing through a cloud of gas-dust while it orbits within our spiral arm of the Milky Way. — Preceding unsigned comment added by Peter Scott Norris (talkcontribs) 22:18, 12 February 2013 (UTC)[reply]

Adding something like that would require reliable sources. And since the claim is quite extraordinary, so should the source be, i.e. several peer-reviewed articles or other contributions by recognised experts in the field. --Stephan Schulz (talk) 22:24, 12 February 2013 (UTC)[reply]
Hmm... galactic winter.... got to find somebody to write a page for me that I can cite.... Gzuckier (talk) 05:36, 13 February 2013 (UTC)[reply]
The idea of the passage of the solar system through interstellar gas clouds is not exactly "extraordinary", I read about it in the 90's in either Discover or Scientific American (probably the former). From what I remember we are expected to enter one in some millennia. Sorry I don't have a reference though. μηδείς (talk) 18:07, 13 February 2013 (UTC)[reply]

Stove vs. oven usage costs

I'm trying to reduce my utility bills, and I'm wondering which of these two costs me more money, assuming both are run for the same duration of time (say, 15 minutes or so):

In other words, should I bake more or boil more? ;) (Yes, I know there are other ways to use these appliances besides baking and boiling.)

SeekingAnswers (reply) 22:50, 12 February 2013 (UTC)[reply]

In response to some additional questions from posters below: I don't know much about ovens, so I can't tell you much more about the oven then that it is an oven. It is built into the kitchen and not portable. Its interior dimensions are roughly a 45-cm or 18-inch cube. —SeekingAnswers (reply) 01:51, 13 February 2013 (UTC)[reply]

In further response to some more questions below: my purpose is to cook food, not to heat the home. —SeekingAnswers (reply) 21:08, 13 February 2013 (UTC)[reply]

You need to give more information before a meaningful answer can be given. What sort of oven is it, for example? And what are you paying for gas/electricity/coal/logs of wood/whatever? On the face of it, the burner is less efficient because it is not enclosed. 86.163.209.18 (talk) 23:17, 12 February 2013 (UTC)[reply]
All else being equal, the oven is more efficient (as explained above) and natural gas is cheaper than electricity per BTU (in most cases). However, there are too many variable parameters to provide a comprehensive answer.   ~:74.60.29.141 (talk) 01:12, 13 February 2013 (UTC)[reply]
The size of the oven also matters. We cooked our roasts, baked cakes, etc in a traditional floor-standing kitchen oven (electric) that has a cooking area capacity about 450 x 450 x 450 mm. As is the norm for such things, it was wired permanently into the house electrical system by the installing electrician as the max draw is far above what can be drawn via a standard plug. Last year I bought a "snack oven" that sits on the bench top and plugs into a standard wall outlet. It has a cooking capacity of 300 x 250 x 200 mm. We found that that is large enough to cook the majority of roasts and cakes we do. It warms up fast, 5 minutes versus 15 minutes for the old oven. Since the elements in either case run flat out during warm up, that alone means a significant saving in electricity. And because of the reduction in surface area, roughly 2.5:1, the electricity consumed during actual cooking is reduced by nearly the same ratio, even though the temperature is the same. Our power company gives a graph of houshold electricity useage over the last year - the graph shows quite a noticeable drop, such that the snack oven paid for itself in a few months at most. Floda 60.230.209.66 (talk) 01:31, 13 February 2013 (UTC)[reply]
The most basic follow-up Q is whether the oven is natural gas or electricity. Electricity generally costs about 3 times as much per heat generated, so that would make it a safe bet to use the natural gas device. If they are both natural gas-powered, then it's more even. Note that the waste heat is useful in winter to help heat the home, while absolutely unwanted in summer, where it can cause you to use the (presumably expensive electric) air conditioner more. In summer, to avoid this, I don't cook on either, but try to eat things like sandwiches, and, for things I do heat, I use the microwave. I tend to use the stove in winter, because the heat and humidity generated are both welcome, then. I only ever use the oven for things which can only be cooked that way, like muffins. StuRat (talk) 04:00, 13 February 2013 (UTC)[reply]
For some reason I thought your question was about heating the house. Both the oven and the burner use natural gas fuel correct? So as such they are both equally efficient since all the energy turns into heat and stays in the house (since you don't vent the exhaust).
However I can't tell you which one will make more heat in the house. The first thing you will want to do is read the nameplate on the stove and see how many BTU the burners and stove are rated for. Once you know that the next question is how long they will stay on. Obviously the burner will stay on as long as you keep it on. The oven has a thermostat - so will cycle. I have no way to know what the duty cycle will be since it depends on the insulation.
And finally, the burner will release all the heat instantly. The oven will keep the heat inside it for longer - it will all eventually be emitted, but over a longer time. Usually this is more efficient since the house will cool faster the hotter it is, so it's better to keep the house at an even temperature, not up then down. However, if the delay in warming causes you to run the heat longer, you might waste some by overshooting. Ariel. (talk) 07:16, 13 February 2013 (UTC)[reply]
I don't want to heat. I want to cook. —SeekingAnswers (reply) 21:47, 13 February 2013 (UTC)[reply]
Ok, so I looked at my oven's nameplate. The burner outputs 9,500 BTU and the stove 16,000. Yours will be different, but I think this is reasonably typical. So in theory this implies that burner uses less energy. But, the oven has a thermostat, so doesn't run all the time. The oven also has insulation. The oven is better at not wasting heat into the air. Boiling water uses a lot of energy, baking uses less (this doesn't apply to frying). So I think the oven will be better - but you will want to turn it off before the food is ready, and let the residual heat cook the food. It will be hard to get the timing right though.
Reality check time: 16,000 BTU = 16 cubic feet of natural gas per hour * 15 minutes = 4 cubic feet. Average retail cost: $10 per 1000 cubic feet. So 15 minutes of maximum oven use costs $0.04 - I suggest looking for savings elsewhere. Ariel. (talk) 07:34, 13 February 2013 (UTC)[reply]
All very good, but the OP hasn't told us whether he/she wants to choose whether to boil or bake to cook food in the least cost way, or to boil or bake to (supplementary) heat the house in the least cost way. To answer the second, we need to know what sort of climate and building the OP lives in. Moderately warm climates usually mean that no house heating is used. Hot climates mean aircon is run to you want the least cooking heat; cold climates mean heating is used so cooking heat makes no differtence. Single unit ground dwellings loost heat much faster than high rise flats. Most people, when they ask questions like this, are just interested in lowering their electricity bills. In average dwellings in moderate climates the heat produced in the kitchen has little effect on comfort and the cost electricity used elsewhere - lighting, aircon, heating, appliances. Floda 121.221.231.38 (talk) 07:54, 13 February 2013 (UTC)[reply]
I don't want to heat. I want to cook. —SeekingAnswers (reply) 21:47, 13 February 2013 (UTC)[reply]
If they really want to save they should microwave if possible. That's more efficient except for heating up liquids where heating on the hob may be better. Dmcq (talk) 14:39, 13 February 2013 (UTC)[reply]
That's only true in summer. In winter, since the waste heat is used to heat the home, thus reducing the furnace load, it shouldn't cost anything more to cook with gas. Cooking with electricity does cost more, though, since electricity typically costs more than gas, per BTU (or your favorite unit). BTW, what's "the hob" ? StuRat (talk) 18:02, 13 February 2013 (UTC)[reply]
The top working surface of a stove. I'm not altogether certain it is a good idea to warm up the kitchen instead of the other rooms. Dmcq (talk) 19:32, 13 February 2013 (UTC)[reply]
I didn't interpret your Q as being about heating the house, but, in case I was wrong, let me add a caution: While unvented gas stoves and ovens don't burn enough oxygen and release enough unburnt gas and carbon dioxide/carbon monoxide to be a problem, normally, when leaving them fully on (with the oven door open) for many hours, with the house windows shut, the air quality can start to suffer. StuRat (talk) 18:08, 13 February 2013 (UTC)[reply]
I don't want to heat. I want to cook. —SeekingAnswers (reply) 21:47, 13 February 2013 (UTC)[reply]

Additional question: how do you tell whether an oven uses natural gas or electricity? In the case of stoves, my understanding is that an open flame indicates natural gas, and those spiral no-flame burners use electricity. What about in the case of ovens? If my stove is natural gas, and my oven sits under my stove, does that automatically mean my oven also uses natural gas, or are they not necessarily correlated? —SeekingAnswers (reply) 21:12, 13 February 2013 (UTC)[reply]

If the top is natural gas the bottom virtually always is too. I've heard of some special hybrid convection ovens that has gas on top, but use electricity and a fan to speed up the cooking but they are very rare. Other ways to tell: Look at the plug. Is it a standard plug or a large one for ovens? When you turn on the oven you should be able to hear the gas ignite it will make a "woosh" or "twump" sound. You can also look inside the oven, there are vent holes and you should be able to see the fire. With electric oven the element is easily visible. I answered your earlier question both ways BTW, cooking and heating. Ariel. (talk) 22:56, 13 February 2013 (UTC)[reply]
The oven is inset into the cabinet and attached to the wall, so I can't actually look behind the oven to see any sort of plug. The oven doesn't have a glass window, so normally I can't actually see inside the oven while it is in operation, but if I open the lid while it is running, I can see some sort of glow or light emanating from gaps at the edges of the bottom surface; however, I can't see any actual flame, though it's possible that my view is just being blocked by the bottom surface. While the oven is in operation, I can hear some sort of quiet semi-hissing which sounds similar to but not identical to when there is an open flame on my stove. Are electric ovens silent? —SeekingAnswers (reply) 00:57, 14 February 2013 (UTC)[reply]
That all sounds like a gas oven to me. Open the door and turn it on, then wait a minute or so and you should hear the fire ignite, and see the light at the same time. That will be the final confirmation. (It will have a delay after turning it on.) In an electric one the glow will get brighter gradually. Ariel. (talk) 01:04, 14 February 2013 (UTC)[reply]
Hmm, that's weird, because then my oven seems to exhibit characteristics of both? After turning on the oven, it is initially silent, but after about 30 seconds to a minute, I hear a click, and then the constant hissing sound begins. However, the glow starts maybe 5-10 seconds before the click, and the glow gets brighter gradually before the click and keeps getting brighter gradually. And when I turn it off, the hissing sound stops first, but the glow continues for a few seconds after the hissing before gradually fading away. —SeekingAnswers (reply) 01:32, 14 February 2013 (UTC)[reply]
Perfect! That is exactly what I was expecting. You have a gas oven. The glow you saw is a glowbar. The way gas ovens work is you want to make sure that you never turn on the gas without ignition. So what you turn on the gas, you actually turn on the glow bar which is heated electrically. When it gets hot enough a mechanical valve opens which lets out the gas. When you turn it off the glowbar cools, which turn of the gas. So the initial glow you saw is the glowbar heating up, and the final glow was the glowbar cooling off. It's interesting that you never actually turn the gas on and off directly - you only control the glowbar. You should be able to remove the bottom of the oven and watch it happen - the bottom panel is usually made to be easy to remove. Or see if you can find a video on youtube. Ariel. (talk) 02:10, 14 February 2013 (UTC)[reply]
Incidentally, the gas ovens I've had have all had a separate compartment at the floor for broiling (heating from above). You have to get your face almost onto the floor (or use a small mirror), but this allows you to see the flames directly, when it's on, with the broiler door open. StuRat (talk) 02:28, 14 February 2013 (UTC)[reply]

February 13

Einstein's theory was wrong

Last sentence of this article Spacetime in General relativity says Einstein's theory was wrong, which surprised me. According to me, this is wrong since this article doesn't contain any reference. What do you think about this? 27.62.78.249 (talk) 02:40, 13 February 2013 (UTC)[reply]

That was just a vandal, and he's been reverted. It's a terrible article though, in any event. Someguy1221 (talk) 02:41, 13 February 2013 (UTC)[reply]
Holy cow, you aren't exagerating... Snow (talk) 02:48, 13 February 2013 (UTC)[reply]
Human beings can believe in terrible ideas. It is no wonder some people believed Eistein's theory was wrong. --PlanetEditor (talk) 03:50, 13 February 2013 (UTC)[reply]
My favourite wacky website Conservapedia has a particular problem with Einstein, see E=mc2? Not on Conservapedia Dmcq (talk) 11:45, 13 February 2013 (UTC)[reply]
It's there under Einstein himself,[29] and has been since at May of 2007.[30] Maybe no one has bothered to write a separate article about it there yet. ←Baseball Bugs What's up, Doc? carrots14:32, 13 February 2013 (UTC)[reply]
OMG! I see why that vandal "improved" our article.
That's incredible...evidently, in the Bible: John 4:46-54, it says that a man visited Jesus saying that his son was dying - Jesus tells him to go home and his son will be just fine. The guy goes back and his household staff tells him that his son got better at 1pm the previous day which was the exact same time he was talking to Jesus! Cool!! So we may conclude that it took zero amount of time for Jesus' cure to reach the child - which means that the cure travelled faster than the speed of light - which Einstein said was impossible! The bible must be right, ergo Einstein was wrong and presumably the Missouri House of Representatives can mandate that it shouldn't be taught in schools. I guess Newton is still OK though...(although he rejected trinitarianism, so you'd better get your kids to learn "f=ma" while they still can - I fear calculus is already a lost cause!)
Then...if that's not enough...we are told that Obama used Einstein's Relativity to "prove" that abortion is OK. The truth of that turns out to be that about 20 years ago, some legal theorist wrote that just as Einstein showed that space-time is curved by gravity, so the landscape of our legal system is changed by the way people interpret the law. He then goes on to use the abortion debate as an example of that changing legal landscape. This paper was published in an obscure law journal that Obama was the editor of. Obama neither wrote nor endorsed the paper, he merely accepted it for publication - the paper made no mention of relativity having anything whatever to do with abortion or even advocated on either side of the abortion debate. But now Einstein is on the religious nut's shit-list right up there with Darwin.
Wow!...just Wow!
SteveBaker (talk) 15:01, 13 February 2013 (UTC)[reply]
Clearly you haven't read much ID literature :-). Einstein has always been on the shit list. One of the greatest difficulties for ID proponents is to explain why, if the world is only a few thousand years old, light from distant stars appears to have been traveling toward us for millions or billions of years. Unless you assume that the light was created "in transit" -- which seems absurd even to ID proponents -- there is no way to make things work without trashing relativity. Looie496 (talk) 16:00, 13 February 2013 (UTC)[reply]
But without a finite speed of light, explaining red-shift would be exceedingly difficult. Understanding why there is a lag in transatlantic telephone calls made via satellite gets tricky...well, all manner of very obvious facts about what we see in our daily lives would be exceedingly hard to explain. These computers we're typing on would be very different beasts if the speed of light were any different than it is. Argh...there is simply no point in chasing it down this particular rabbit-hole. Demolishing these arguments is like shooting fish in a barrel. SteveBaker (talk) 20:41, 13 February 2013 (UTC)[reply]
Devils advocate: If you really wanted, you could explain redshift and propagation delay without a finite speed of light by assuming luminiferous aether exists (with some adjustments of course, perhaps assuming that the aether flows under gravity). Ariel. (talk) 23:05, 13 February 2013 (UTC)[reply]
I see Conservapedia now have a E=Mc2 article. It was set up with the comment 'E=mc2 is a meaningless statement in physics that purports to relate light to matter. In fact, no theory has successfully unified the laws governing mass (i.e., gravity) with the laws'. So perhaps to say that they cover it is about equivalent to saying we cover the government conspiracy over 9/11. Dmcq (talk) 20:48, 13 February 2013 (UTC)[reply]
"Simply put, E=mc² is liberal claptrap." --PlanetEditor (talk) 02:26, 14 February 2013 (UTC)[reply]

How many times thicker is the dermis than the epidermis?

At the same spot. Sagittarian Milky Way (talk) 02:59, 13 February 2013 (UTC)[reply]

Well, the thickness of dermis and epidermis are not the same at every spot of your body; the thickness varies. The epidermis is 0.05 mm ( eyelids) to 1.5 mm (palms and soles) thick at different places. On the other hand, the dermis is 0.3 mm to 4 mm thick at different spots. As a rule of thumb, the dermis is 10 times thicker than epidermis at the same spot. But this thickness can go further in the range of 10 to 40 times at some places. --PlanetEditor (talk) 03:09, 13 February 2013 (UTC)[reply]
(edit conflict)
"The dermis is 10 to 40 times thicker than the epidermis.""[31]
~:74.60.29.141 (talk) 03:13, 13 February 2013 (UTC)[reply]
But this "rule" does not apply uniformly. For example, according to the Wikipedia article Skin, the skin on the palms is 4 mm thick and the epidermis on the palms is 1.5 mm thick. So the dermis on the palms is 4 - 1.5 = 2.5 mm thick. So the dermis on the palms is 1.6 times thicker than the epidermis on the palms. How is it 10 times thicker? --PlanetEditor (talk) 03:41, 13 February 2013 (UTC)[reply]

Value chain of SLES (Sodium Lauryl Ether Sulfate)

Need to know the value chain of SLES (Sodium Lauryl Ether Sulfate) with all the respective % , sources, Process and cost indication details — Preceding unsigned comment added by 170.252.219.1 (talk) 04:24, 13 February 2013 (UTC)[reply]

Please do your own homework.
Welcome to the Wikipedia Reference Desk. Your question appears to be a homework question. I apologize if this is a misinterpretation, but it is our aim here not to do people's homework for them, but to merely aid them in doing it themselves. Letting someone else do your homework does not help you learn nearly as much as doing it yourself. Please attempt to solve the problem or answer the question yourself first. If you need help with a specific part of your homework, feel free to tell us where you are stuck and ask for help. If you need help grasping the concept of a problem, by all means let us know.--Jayron32 06:06, 13 February 2013 (UTC)[reply]

Homo sapiens subspecies

I was just going through the article Elephas maximus and found the basis of their subspecies classification:

  • Elephas maximus maximus: shoulder height between 2 and 3.5 m, 19 pairs of ribs, skin color is darker than of indicus and of sumatranus
  • Elephas maximus indicus: shoulder height between 2 and 3.5 m, 19 pairs of ribs, skin color is lighter than of maximus with smaller patches of depigmentation, but darker than of sumatranus
  • Elephas maximus sumatranus: shoulder height between 2 and 3.2 m, 20 pairs of ribs, skin color is lighter than of maximus and indicus with least depigmentation

Then I read the article Subspecies. According to Subspecies#Criteria,

Members of one subspecies differ morphologically or by different coding sequences of DNA from members of other subspecies of the species. Subspecies are defined in relation to species.

This means a species may have two or more subspecies which are:

  • either morphologically different, or
  • genetically different, or
  • both.

Now I have a thought. If elephants can be divided into different subspecies based on morphological differences, why can't humans be classified into different subspecies given that humans have vast morphological as well as genetic differences. I'm not talking about the extinct Homo sapiens idaltu or the extinct Homo sapiens neanderthalensis, I'm talking about extant Homo sapiens. Also, it has nothing to do with scientific racism which is the scientific manifestation of the political belief of racial superiority. Differentiating humans into different subspecies should be scientific (just as in the case of Asian elephants) and does not claim one subspecies is superior to the other, it is just morphological and genetic differences. So I'm curious to know why humans have not been classified into subspecies. --PlanetEditor (talk) 09:20, 13 February 2013 (UTC)[reply]

In modern times with modern techniques, a new classification of subspecies would only be accepted if it had at least a hypothetical phylogenetic basis. And although you can classify people into groups based on genetic similarities, it's very difficult to form a monophyletic groups of humans that can be distinguished from other such groups by any significant, unique characteristics. You can certainly group people by race, and these groupings will have some genetic and morphological traits they share more often with eachother than with outgroups, but it will not be taxonomically valid. And that's not to even mention all the interbreeding that takes place. Read more at race and genetics. I know you didn't ask specifically about race, but that's the most on-topic article. Someguy1221 (talk) 09:33, 13 February 2013 (UTC)[reply]
Thanks for the explanation. --PlanetEditor (talk) 10:57, 13 February 2013 (UTC)[reply]

Homo sapiens sapiens, Homo sapiens neanderthalensis. μηδείς (talk) 18:02, 13 February 2013 (UTC)[reply]

Homo sapiens idaltu. RNealK (talk) 22:23, 13 February 2013 (UTC)[reply]

What is the latest on olive oil and antioxidants? Are they supposed to be good or bad for you now?

What is the latest on olive oil and antioxidants? On balance of the best evidence available, are they supposed to be good or bad for your health? --173.49.81.79 (talk) 13:36, 13 February 2013 (UTC)[reply]

On the balance of what? Insofar as you use olive oil in stead of other oils, it is probably better for you than those other oils, but in general it is best to keep overall fat consumption low (not zero, but probably lower than the average western diet). So olive oil is both good (better than other, highly refined oils) and bad (too much fat in a diet of any sort can be problematic). And there are several "antioxidants"(which is a marketing term that has become almost meaningless due to its overuse) which are not only good, but are vital, such as Vitamin C. --Jayron32 13:46, 13 February 2013 (UTC)[reply]
Too much anything (fat, carbohydrates, protein, vitamins, mineral, water, oxygen) is bad for health. What is important is to check the daily total calorie consumption. A popular misconception exists about carbohydrates. Carbohydrates are not essential nutrients; although dietary fibers are important for health. Carbohydrates only provide energy, and processed carbohydrates (high glycemic load) and fruits (due to presence of fructose) do a lot of harm. --PlanetEditor (talk) 14:14, 13 February 2013 (UTC)[reply]
Olive oil is low in saturated fat (commonly called "bad fat" linked with coronary heart disease), high is MUFA (good fat), and moderate in PUFA (another fat which is both "good" and "bad"). A high PUFA consumption is detrimental to health, and there have been a lot of noise made about the effects of saturated fat. So olive oil is an ideal choice for cooking and salad dressing, it will give you the good fat MUFA, and limit your saturated fat consumption. Another oil that you can chose is canola oil. --PlanetEditor (talk) 13:54, 13 February 2013 (UTC)[reply]
Antioxidants are essential. But a study shows excess intake of vitamin c can reduce the benefits of exercise. However in that study the volunteers took 1000 mg vit. C per day, which is a very high dose and well above the RDI. --PlanetEditor (talk) 14:02, 13 February 2013 (UTC)[reply]

See also:

"People have evolved eating food and adapted to the quantities of bioactive compounds in food, so given a choice, obtaining nutrients and bioactives through food is always the best choice. We are learning that for many bioactives, it is not a single substance but the complex milieu in food that provides the benefit."
—John Finley, National Program Leader in Human Nutrition for the Agricultural Research Service of the U.S. Department of Agriculture
CONCLUSION:   Although phenols from olive oil seem to be well absorbed, the content of olive oil phenols with antioxidant potential in the Mediterranean diet is probably too low to produce a measurable effect on LDL oxidisability or other oxidation markers in humans. The available evidence does not suggest that consumption of phenols in the amounts provided by dietary olive oil will protect LDL against oxidative modification to any important extent.
Vissers, MN (2004 Jun). "Bioavailability and antioxidant effects of olive oil phenols in humans: a review". European journal of clinical nutrition. 58 (6): 955–65. PMID 15164117. {{cite journal}}: Check date values in: |date= (help); Unknown parameter |coauthors= ignored (|author= suggested) (help)
~: 15:45, 13 February 2013 (UTC):[modified:74.60.29.141 (talk) 16:00, 13 February 2013 (UTC)][reply]
Four other things to be worried about with olive oil:
1) Not heat stable, so don't cook with it.
2) UV light damages it, so get in in a dark or opaque container.
3) Often old/rancid. Look for a harvest date within a year or so, and don't buy if they omit that date (a "best by" date doesn't cut it).
4) Often adulterated with cheaper oils. StuRat (talk) 15:42, 13 February 2013 (UTC)[reply]
Re, #1, above:
"When heated, olive oil is the most stable fat, which means it stands up well to high frying temperatures. Its high smoke point (410ºF or 210ºC) is well above the ideal temperature for frying food (356ºF or 180ºC). The digestibility of olive oil is not affected when it is heated, even when it is re-used several times for frying."
International Olive Oil Council (IOOC) –[Per: Heating Olive Oil; The Olive Oil Source]
However:   "Heats between 320-374˚F (160-190˚C) are most often used to determine the changes that occur in extra virgin olive oil due to heating. Studies on this subject repeatedly show that heats as low as 320˚F (160˚C) can substantially damage the phenols in olive oil."
"Heat damages extra virgin olive oil". The World's Healthiest Foods. The George Mateljan Foundation. Retrieved 13 February 2013.
~: 16:13, 13 February 2013 (UTC) [Last modified:74.60.29.141 (talk) 19:07, 13 February 2013 (UTC) Did not "edit others' posts"[reply]
Unlike articles, you're welcome to dispute the statements of others here, but not to edit their posts (since it makes it look like they said something which they did not). Also, note that your first source also says "High quality extra virgin olive oils (with low free fatty acids) have a high smoke point. They are an excellent choice, but an expensive one. Mass produced, low quality olive oils have a much lower smoke point". StuRat (talk) 17:53, 13 February 2013 (UTC)[reply]

My gut feeling from the 1990s on has been that talking about "antioxidants" is the biggest bunch of hooey in science. Biology doesn't know theory, and cells aren't test tubes. You don't know what a substance will do until you test that substance. It is true that hypothesizing that antioxidants are good for the body has led to the identification of foods (including olive oil) that show benefits in some studies, but the same would probably be true of any means by which you create a long list of traditional human dietary items. In any case [32] leads me to [33] which cites some beneficial effects on rat coordination. However, bear in mind that they are feeding those rats a very large amount of oil, so this is much like the infamous "causes cancer in rats" sort of study that people like to complain about. It is impossible to say whether rats given a wide, natural choice of dietary items rather than a fixed laboratory diet would see the same benefit from adding phenolic compounds to it. Even so, well ... olive oil was invented by Athena, and Crisco was invented by Proctor & Gamble. Given a choice, I'd say go with the good stuff. (Well, except, I shouldn't be giving medical advice, especially when I admit I have no idea, that is) Wnt (talk) 16:04, 13 February 2013 (UTC)[reply]

This link says use only extra virgin oil, which is for salad dressing. But for cooking, I have to buy something other than extra virgin. I use an olive oil the nutrition label of which says it contains 2 g saturated fat, 10 g MUFA, 2 g PUFA, and 0 trans fat per 10 g. The Guardian article says other categories—"pure" or "light" oil, "olive oil" and "olive pomace oil" – have undergone chemical refinement. Is "chemical refinement" something to be worried about when there is no trans fat present? --PlanetEditor (talk) 16:14, 13 February 2013 (UTC)[reply]

KWh problem

I have a problem where a fridge has a COP of 1.4. A 20W light in the fridge is switched on when the fridge door is open which is for 31,317,000 seconds of the year. I require the energy consumption of the fridge (for the light bulb only so ignoring energy consumption to power fridge itself) in KWh/year. I know from my calculations that the energy the bulb requires in a year is 20x31,317,000 which is 626340000J. The energy the fridge can supply is 20/1.4 which is 14.29J. This would be per second I think. I'm lost from here because of the KWh/yr. I don't understand this unit and I don't know how to get the energy consumption of the fridge for the bulb in KWh/yr. Clover345 (talk) 16:42, 13 February 2013 (UTC)[reply]

Are you sure about this setup? Your time value is equal to the full number of seconds in a year, so you are assuming that the door is open all the time. Is that really what you want to do? Regardless of the answer, converting to Joules here does not help you. Basically your problem is to convert from Watt-seconds to Kilowatt-hours -- the "/yr" part simply indicates that you should do this on a one-year basis. Looie496 (talk) 16:55, 13 February 2013 (UTC)[reply]
I couldn't find the TLA "COP" in Wikipedia (unless you meant Coefficient of performance which doesn't apply to the light bulb), so I'm not sure where the 1.4 comes in. I agree with Looie496 that the joule isn't the simplest unit to use here, but, if you insist, then 20W is 20 joules per second, and one kWh is: 1000 (joules per second) times 60 (seconds in a minute) times 60 (minutes in an hour) times 24 (hours in a day) joules. Alternatively, if you look at the Kilowatt hour article you will see that one kWh is equivalent to 3.6 million joules. Leaving the fridge door open for a full year with the fridge switched on will waste a lot more energy than that used by the light bulb. Dbfirs 17:29, 13 February 2013 (UTC)[reply]
Leave out Dbfirs' last "times 24" or you'll be dealing in "kilowatt days" Rojomoke (talk) 17:41, 13 February 2013 (UTC)[reply]
Oops! Brain slipped out of gear! Dbfirs 20:58, 13 February 2013 (UTC)[reply]
Don't forget that the bulb dumps waste heat into the refrigerator - which it then has to remove - so the actual amount of energy consumed if the light inside fails to turn off when you shut the door is going to be at least twice what the bulb consumes...probably much more. SteveBaker (talk) 20:36, 13 February 2013 (UTC)[reply]
Your estimate is far too high. Typically it takes %25 of the input energy to cool that amount. So a 40 watt light will consume 10 watt in order to cool it. Maybe slightly more because the fridge is cold, but certainly not "twice what the bulb consumes". Ariel. (talk) 01:07, 14 February 2013 (UTC)[reply]
I think I recall this problem from my 'Physics 101' exam. -Which doesn't necessarily mean that I recall the answer.  ;)  ~:74.60.29.141 (talk) 21:09, 13 February 2013 (UTC)[reply]
COP (coeficient of performance) is the ratio of heat removed to electrical energy consumed and applies to any form of refrigeration - airconditioners, fridges, thermo-electric beer cooler, etc. To get the electricity consumed in kW.hr due to the 20 W light, you need to multipy 20 watts by 1 plus 1/COP, and then multiply by the total door open time in hours. You need to add 1 because the light draws power directly, as well as causing the refigeration to draw more power. However, the COP given by the OP is way too low. Typical kitchen fridge COP's will be in the range 3 to 4. Some very small units can be as low as 2. Wickwack 124.178.132.174 (talk) 00:17, 14 February 2013 (UTC)[reply]

Where can I find real video of the moon orbiting the Earth from perspective outside Moon's orbit?

A few years ago, I remember watching a video of the Moon orbiting the Earth from a perspective outside the Moon's orbit, but I can't seem to find it. This is actual footage, not an animation. I don't remember all the details, but obviously there can't be too many videos of the Moon orbiting the Earth from this perspective. The video was short, maybe a minute or two, and time-elapsed. I'm guessing the video was taken from a satalite orbiting the Sun, but I could be mistaken on that. Can anyone help me out here? A Quest For Knowledge (talk) 16:49, 13 February 2013 (UTC)[reply]

Was it this 2008 video of the transit of the Moon in front of Earth, as seen by the Deep Impact probe? TenOfAllTrades(talk) 17:05, 13 February 2013 (UTC)[reply]
For bonus points, I'll also throw in this small collection of still images taken by other missions. The Voyager image at the bottom of the page was the very first of its kind. TenOfAllTrades(talk) 17:11, 13 February 2013 (UTC)[reply]
@TenOfAllTrades: Yep, I'm pretty sure that was it. Thanks! A Quest For Knowledge (talk) 17:21, 13 February 2013 (UTC)[reply]

Coefficient of Performance

Are my calculations here correct?

My air con unit has a temp of 5C in its coils, and the temp outside is 25C, the rate of heat transfer from the coil to the refrigerant is 5000kj/hour, and the power input required is 1250 kj/hour.

I've calculated the Coeffecient of Performance as 5000/1250 = 4 and the maximum coefficient of performance if the system were reversible as 1/(298/278)-1 = 13.9.

I've also calculated what it's Coefficient of performance would be as a heater if I reversed the unit as 1/1-(278/298) = 14.9. This is the max Coefficient of Performance as a heater so in reality it would be 4(from the Coefficient of performance as a air con) + 1 =5. 1 is from 14.9 -13.9. Clover345 (talk) 17:51, 13 February 2013 (UTC)[reply]

I see nothing wrong with your math, except I don't know where you got the 298/278 numbers. For heating COP you want: (5000+1250)/1250 = 5. BTW this is a pretty efficient A/C, I don't believe any units of this size, and still this efficient are available on the market. (This efficiency is usually only available for much larger sizes than this.) Ariel. (talk) 18:30, 13 February 2013 (UTC)[reply]
Also "the rate of heat transfer from the coil to the refrigerant" - for a system like this you actually want the rate from inside air to outside air. There are two heat transfers in this system, so don't just calculate one of them. Ariel. (talk) 18:31, 13 February 2013 (UTC)[reply]
The 298/278 are the temps in kelvin. And can you elaborate on the second system I should calculate? Thanks. Clover345 (talk) 19:06, 13 February 2013 (UTC)[reply]
Heat transfer from the room to the cold coils, and from the hot coils to the outside air. If you are using temperature to do your calculations you have to do it twice (you started your question by mentioning temperatures). If you just look at net flow then both will be the same of course, except that the hot side also deals with waste heat. Ariel. (talk) 23:09, 13 February 2013 (UTC)[reply]

basic theoretical question (no application)

Hi,

I have a basic theoretical question that has no application. Is a glider, like a sailboat, able to "glide" against wind (when looking at ground coordinates, I mean). What are the theoretical constraints? What I mean is, theoretically if you are at an altitude, A, and have a glider of mass m and wing area w that is willing to lose y meters of height in order to go straight into the direction from which wind is coming from at a velocity V - will that be possible? Or, normally, will a glider be unable to perform this overall action. If it can perform the action overall (including via crisscrossing) what is the average theoretical limit of the speed it can gain (wrt to ground) in directly same direction wind is coming from? This is a point of curiosity only and has no intended applications at present, but is intended to help me understand basic ralationships. Thanks. 178.48.114.143 (talk) 18:41, 13 February 2013 (UTC)[reply]

No. A glider can (for a while) steer into the wind and make progress by losing speed and/or altitude - or by maintaining altitude in a rising air stream...but it can't "tack" into the wind like a sailing ship. To do that the sailing ship is using the resistance of the water against the flow of the air. A land-yacht does is using friction against the ground with its' wheels. The glider doesn't have anything to push against, so it can't do that. A glider that could fly up-wind would be a perpetual motion machine - and those don't work! SteveBaker (talk) 20:32, 13 February 2013 (UTC)[reply]

Could you or someone specify what "for a while" is theoretically, for different wind speeds, altitude a, mass m, wing area w, and so forth. Am I supposed to just take the potential energy that the glider has due to its altitude, and convert that into forward motion directly (i.e. in newtons)? i.e. I can theoretically give a glider as much forward thrust as it loses in kinetic energy from losing energy? In that case would I simply subtract the speed of the wind from that to see if it can (even theoretically) move forward with respect to ground when faced with that wind, losing altitude to do so? My suspicion is there is a limiting rate, thus for any combination of the above there is a wind you cannot fight for any amount of time as you just can't convert altitude to forward motion fast enough. I'd like to know the equation I can use to calculate this, if possible. This is meant to increase my theoretical understanding only. Thanks for any help. 178.48.114.143 (talk) 21:36, 13 February 2013 (UTC)[reply]

Have you read Polar curve (aviation)? --catslash (talk) 22:39, 13 February 2013 (UTC)[reply]
If you want to increase your theoretical understanding, please start by reading our article on gliding flight. Looie496 (talk) 22:46, 13 February 2013 (UTC)[reply]

Could any engineer device a toilet operation contingency plan for cruise-ships?

Seen this? http://www.cnn.com/2013/02/13/travel/cruise-ship-fire/index.html?utm_source=feedburner&utm_medium=feed&utm_campaign=Feed%3A+rss%2Fcnn_topstories+%28RSS%3A+Top+Stories%29

Whenever I get on my first cruise ship, I would like to be reassured that the ship's sanitary systems have some kind of contingency plan so that if the main source of power fails, there will be some way to keep the toilets running anyhow; a fail-safe.

I wouldn't mind if the toilets even opened a "contingency valve" that releases the waste directly to the waters (but ONLY during such emergencies like the Carnival Triumph's) as that emergency would make the waste-release the lesser of our concerns.

But what is preventing that from happening? And what would it take to invent a contingency system that will keep a cruise ship's toilets working anyhow during this kind of emergency? How hard is it to invent, what would the cost be, how much extra $ would that be passed on to passengers in terms of ticket prices, and how many years off do you suppose such inventions are?

Remember that necessity is the mother of invention! There's very much a necessity for this happening lately! Thank you. --70.179.161.230 (talk) 19:28, 13 February 2013 (UTC)[reply]

Ships use vacuum toilets to keep pipe size and weight down and to assure operation when the ship is in motion. Buildings, which don't move, operate entirely by gravity, but need bigger pipes and more-or-less direct runs to a drain or sump, and aren't suitable for seagoing use on a large scale. I'm not sure what the pertinent maritime law is, but I doubt that it allows for an emergency sewage system on large vessels: they must operate under all circumstances. It seems to me that backup power is needed to assure that the vacuum system works: it apparently wasn't a priority in this case, or it failed or was damaged in the fire. Acroterion (talk) 19:33, 13 February 2013 (UTC)[reply]
Buckets, gunwale, lots of wet wipes... passengers might raise a ruckus, but it's simple, it works and it is cheap. And for a short term emergency solution it's good enough. WegianWarrior (talk) 20:24, 13 February 2013 (UTC)[reply]
The solution is to make sure you don't totally lose power - and that requires generators and fuel for those generators to be stored someplace OTHER than where the main generators and their fuel is stored. If it hadn't been the toilets backing up - or the food running out - or the A/C not working, there would have been something else. A ship like this simply can't function without adequate power...and a design that allows it to lose all power from a single point of failure (like a fire in the engine room) is a poor design. They needed to spend the money to keep a couple of days of reserve power and not on some complicated solution for zero-powered toilets. That sounds tough to do though. SteveBaker (talk) 20:54, 13 February 2013 (UTC)[reply]
They could have a supply of Porta-John™ folding portable toilets: "Ideal for special events, emergencies, disaster relief and military use".[34]  (They can be shipped/stored 12 per pallet)[35] ~E:[edit]74.60.29.141 (talk) 22:21, 13 February 2013 (UTC)[reply]
This is about the sixth total power outage on such a cruise ship in recent years. The owners seem to register the ships in countries which do not require publishing the results of root-cause investigations, so it is hard to find out why having two separate engine rooms does not provide the expected redundancy when one diesel generator catches fire. Surely a fire in one engine room does not lead to spraying water or foam in both rooms. If it spreads to the 6600 volt switchboard and knocks it out, that might be an explanation. Redundant switchboard design (ring buses) at increased cost might be an option. Complete loss of propulsion in a storm or with the wind driving the ship towards the rocks might doom the 4000 aboard in some future failure. If i somehow get the money for a cruise, I'll go somewhere and stay in nice hotels instead. Edison (talk) 01:01, 14 February 2013 (UTC)[reply]
  • The usual procedure while fishing is to squat over an empty bait bucket and toss it over the side, or just aim over the side if you're equipped for that. Unless a bucket's unsuitable for Hyacinth?
If she was put in charge of the bucket brigade she would perform... her duty, spectacularly I am sure, although she will be repeatedly wishing she could be providing fancier chamber pots for the more prestigious quests. :-) --Modocc (talk) 01:12, 14 February 2013 (UTC)[reply]
They don't seem to have much of an emergency plan, do they ? I can think of many ways to improve things:
1) Bring ships alongside and connect cables to provide electricity.
2) Offload passengers onto other ships.
3) Deliver many Port-a-Potties.
4) During construction, they could have covered the sides and top of the ship with solar panels, which should hopefully be enough to operate the toilets during daylight, at least. When everything is working, they could also supply extra power. StuRat (talk) 00:51, 14 February 2013 (UTC)[reply]
There isn't really a "top" available to cover - that's the deck, and people walk on it. The sides would get hit with saltwater which would destroy the solar panel in short order. Ariel. (talk) 01:31, 14 February 2013 (UTC)[reply]
Put a clear lacquer over them, both to protect from salt-water and footsteps. The panel would need to be cleaned regularly, but they clean those areas anyway. StuRat (talk) 01:39, 14 February 2013 (UTC)[reply]
I wonder how much tidal power wave energy a cruise ship could extract from waves lapping up against it (without spending too much on the backup system that is). Wnt (talk) 01:35, 14 February 2013 (UTC)[reply]
It's a customer problem, not an engineering problem. Any reasonably competent electrical engineer can design a redundant power system that makes a ship-wide power outage about as likely as the ship being hit by a meteor. The customers have not demanded that, but they have demanded lower ticket prices, and so they are getting what they want. --Guy Macon (talk) 01:36, 14 February 2013 (UTC)[reply]
I wonder if the customers can sue for pain and suffering ? Then again, if they were sailing to a third-world nation, how wonderful to have a taste of life in the third world for themselves: no food, no light, no electricity, no running water, and sewage everywhere ! StuRat (talk) 01:43, 14 February 2013 (UTC)[reply]
I don't see it as a customer problem. You can't expect tourists & holidaymakers, most of whom are probably making a one-time trip, to understand anything about what it takes to run a ship properly. I don't know anything about that particular ship and what happened to it. But as a professional Engineer who has investigated incidents elsewhere, and performed maintenance audits, I can say that these sorts of problems are almost NEVER due to deficiencies in design. It is sufficiently easy, and quite the norm, in ships, hospitals, and other large projects where life must be protected, to design and build in redundancy in power generation and electrical distribution. And, if the operating Engineers and maintenance staff are left alone to do their jobs, mostly they will maintain that designed in redundancy, becaue hey understand its' importance. The problem arises with non-technical management at the top - business managers, promoted matrons, and the like in hospitals, captains and admin officers in ships, who came up from the business or navigation career path (or, in one case I know about, the captain was a complete fool who passed exams and got promoted up out of the ship's security team!) Upon learning that a) the ship/hospital/whatever has a dual power distribution, and b) one of the duals has failed, but the ship/hospital can still function with only one, they tell the engineers "don't fix it yet - the budget is tight / we need to leave port / whatever.) And then the one remaining system fails. Ratbone 121.221.231.100 (talk) 02:24, 14 February 2013 (UTC)[reply]